Nervous system. Spinal cord. Spinal ganglion.

After amputation of the left top finiteness of the patient long time felt a strong pain in it. What mechanism of formation of painful unhealthy sensations is most probable in this case? A. Fantomic * B. Reflectory C. Kausalgic D. Hypoproduction of encephalines E. Hyperproduction of encephalines

In a histological preparation the organ which will consist of grey and white substance is determined. The grey substance settles down in the center and will consist from radicular, associative neurons. Name body for which the given morphological attributes are characteristic. A. Big hemispheres B. Brain C. Cerebellum D. Spinal ganglia E. Spinal cord*

In a histological preparation parenchyma of organ it is submitted by nervous fibers in which are determined pseudo-unipolar neurons. Neurons’ bodies are covered by glial and connective shalls, placed by groups. Named organ that has given morphological attributes. A. Epiphysis B. Spinal cord C. Spinal ganglia * D. Brain E. Cerebellum

At a mechanical trauma the site of a peripheral nerve in length more than 10 sm is damaged injured. In result impellent activity of the top finiteness is broken. Allotransplantation of criopreserved nerve is offered to the patient. What glial cells will take part in regeneration and maintenance of trophic the damaged injured site? A. Microglial cells B. Fibrous astrocytes C. Neurolemmocytes * D. Ependimogliocytes E. Leucocytes

As a result of a trauma a patient has damaged anterior roots of spinal cord. What structures have been affected? A Axons of motoneurons and axons of neurons of lateral horns * B Central processes of sensitive neurons of spinal ganglions C Peripheral processes of sensitive spinal ganglions D Axons of neurons of lateral horns E Dendrites of neurons of spinal ganglions

A ventral root of spinal cord was damaged as a result of a trauma. The following processes of the following neurons were damaged: A Axons of motor neurons* B Dendrites of motor neurons C Axons of sensory neurons D Dendrites of sensory neurons E Dendrites of internuncial neurons

1 As a result of a trauma a patient has damaged frontal spinal roots. What structures are likely to be affected? A Axons of the motoneurons and axons of the lateral horn neurons* B Central processes of the sensory neurons of the spinal ganglions C Peripheral processes of the sensory neurons of the spinal ganglions D Axons of the lateral horn neurons E Dendrites of the spinal ganglion neurons

A patient had a trauma that led to the injury of front spinal roots. Denote the damaged structures: A *Axons of motoneurons and lateral horn neurons B Central processes of spinal ganglion neurons C Peripheral processes of spinal ganglion neurons D Axons of lateral horn neurons E Axons of motoneurons

Nervous system. Cerebrum. Cerebellum.

On preparation organ of nervous system which contains grey and white substance is submitted. The grey substance is placed on periphery. Neurons form three layers: molecular, ganglionar and granular. What organ is on preparation? A. Brain cortex B. Spinal cord C. Spinal ganglia D. Pons E. Cerebellum cortex*

In a histological preparation the cut of precentral girus of the hemispheres is submitted. Specify, what layers are most advanced in this zone? A. Molecular, pyramidal, ganglionar B. Molecular C. External and internal granular D. Molecular and the layer of polymorphic cells E. Pyramidal, ganglionar and a layer of polymorphic cells *

The alcoholic intoxication, as a rule, is accompanied by infringement of coordination of movement and balance, as a result of damage of structural elements of a cerebellum. Function of what cerebellum’s cells is broken first of all? A. Stellate cells B. Pyramidal cells C. Fusiform cells D. Pear-shaped cells* E. Granular cells

In histological section, impregnated by salts of silver, determined the cerebellum cortex which contains pear-shaped, bascet-shape, stellate neurons, cells-grains. Named neurocytes which are part of the ganglionar layer. A. Pear-shaped* B. Stellate, pyramidal C. Cells - grains, large stellate D. Large stellate and bascet-shape E. Bascet-shape, small and large stellate

In a histological section the body which will consist of grey and white substance is determined. The grey substance settles down on periphery and has 6 layers: molecular, external granular, pyramidal, internal

2 granular, ganglionar and the layer of polymorphic cells. Determine structure to which possesses the given morphological attributes. A. Cerebellum B. Brain* C. Bone marrow D. Spinal ganglia E. Spinal cord

One of the parts of central nervous system has stratified arrangement of neurocytes. Among them there are cells of the following shapes: stellate, fusiform, horizontal, pyramidal. What section of central nervous system is this structure typical for? A Cortex of cerebrum * B Spinal cord C Cerebellum D Medulla oblongata E Hypothalamus

Microscopic examination of a CNS body revealed the gray matter with three layers of neurons, namely molecular, ganglionic and granular layer. What are the neurons constituting the second layer? A. Pirirform* B. Basket C. Small stellate D. Large stellate E. Granule cells

In the newborn child tumor of the spinal cord gray matter was found. What embryonal germ is connected with this? A. Mantle zone* B. Ependymal zone C. Nervous cord D. Ganglionar plate E. Mesoderm

As a result of a trauma a patient has damaged posterior roots of the spinal cord. What structures have been affected? A. Sensory neurons* B. Motor neurons C.Interneurons D. Dendrites of neurons of spinal ganglions E.Central processes of sensitive neurons of spinal ganglions

In the microspecimen of the cerebral cortex large pyramidal cells are reveled. These cells are most significant sign of the cerebral cortex. Discover of these cells is associated with next name: A. Betz* B. Goldgi C. Lenoshek D. Nissl E. Kakhal

In the histological specimen was represented precentral gyrus of the cerebral cortex. Indicate which layers development best in this layer? A. Pyramidal, ganglionar and polymorphic cells layer (external and internal pyramidal and polymorphic cells layer) * B. Molecular

3 C. External and internal granular D. Molecular and polymorphic cells layer E. Molecular, pyramidal and ganglionar (external and internal pyramidal, molecular)

One of sections of central nervous system has layerwise arrangement of neurocytes. Among them there are cells of the following forms: stellate, fusiform, horizontal, pyramidal. What section of central nervous system is this structure typical for? A Cortex of cerebrum* B Spinal cord C Cerebellum D Medulla oblongata E Hypothalamus

In a specimen that was coloured by method of silver impregnation some piriform cells with 2-3 evident dendrites were found. What structure is being analysed? A Cerebellar cortex* B Spiral organ of middle ear C Retina D Cerebral cortex E Spinal ganglion

For a histological investigation we took a sample from the cerebellar hemisphere in which we study the general morphology of nerve cells. What method is appropriate for staining the specimen? A. Impregnation of silver* B. Hematoxylin and eosin C. Sudan and safronin D. Orsein E. Iron hematoxylin

In a specimen of the cerebral cortex, impregnated with silver nitrate, we identify pyramidal shaped cells with a perikaryon diameter up to 120 microns. Which layer of the cortex forms these cells? A. Ganglionic layer* B. Pyramidal layer C. Molecular layer D. External granular layer E. Internal granular layer

A region of the central nervous system has a layered arrangement of neurons among which are cells of such forms as: stellate, spindle, horizontal and pyramidal. Which region of the nervous system corresponds to such structure? A. Cerebral cortex* B. Spinal cortex C. Cerebellum D. Medulla Oblongata E. Hypothalamus

The motor centers of the cerebral cortex has a characterizing agranular type structure. Which layers are the most developed in these centers? A. Pyramidal, granular and polymorphic cells* B. External and internal granular layer and layer of polymorphic cells C. Molecular and pyramidal D. Molecular, external and internal granular layer E. Pyramidal, molecular layer of polymorphic cells

4 On a section of the cerebral hemisphere, we observe the cortex in which we can distinguish nuclei of many cells. These cells include the following except: A. Neutrophils* B. Neurons C. Astrocytes D. Oligodendrocytes E. Microglial cells

In a specimen of an organ of the CNS, impregnated with silver are revealed large fusiform cells located in a single row. From the apices arise 3-4 processes that branch in a single plane. What cells are these? A. Purkinje cells* B. Betz cells C. Granular cells D. Stellate cells E. Basket cells

Which of the below listed afferent tracts reaches the cerebral cortical hemisphere bypassing the thalamus? A. Olfactory tract* B. Vestibulocochlearis C. Exteroreceptor D. Optic tract E. Extracorticospinal tract

Alcohol intoxication is accompanied by violations of motor coordination and balance as a result of damaged to the structural elements of the cerebellum. The function of which cells of the cerebellum are damaged in the first place? A. Fusiform cells* B. Granular cells C. Basket cells D. Stellate cells E. Horizontal cells

In the postoperative period, a patient has no intestinal peristalsis. Dysfunction of which section of the nervous system is mostly like affected? A. Cerebral cortex* B. Lateral horn C. Ventral horn D. Dorsal horn E. Nucleus of hypothalamus

A specimen stained with silver salt impregnation method in which can be seen fusiform cells with well expressed 2-3 dendrites, which rise to the surface is being investigated. Name the specimen? A. Cerebellar cortex* B. Spiral organ of corti C. Retina D. Cerebral cortex E. Spinal ganglion

A specimen of a dead human brain was used for forensic medical examination. In the section of the central gyrus of the cortex, pyramidal layers were weakly expressed, small amount of neurons and number of glial cells are increased? What could the victim be suffering? A. Limb's paralysis* B. Deafness

5 C. Blindness D. Disorder of smell E.

In an experiment of the brain cells of animals, one damaged the cells of granular layers of the cortex. what function of the cortex will be destroyed.? A. Sensory* B. Motor C. Trophic D. Association E. Protection

The efferent system of the cerebellum forms: A. Axons of purkinje cells* B. Axons of granular cells C. Mossy fibres D. Climbing fibres E. Dendrites of purkinje cells

Vegetative nervous system.

Long stay in conditions of high temperature caused thirst in the person. The signal system from what receptors, first of all, has caused its development? A. Sodium receptors of hypothalamus B. Osmoreceptors of hypothalamus* C. Baroreceptors of aorta’s arch D. Glycoreceptors of hypothalamus E. Osmoreceptors of liver

During surgical operation nerve was cut. After some period of time it regenerated. Call cell which provide its regeneration. A. Neurolemmocytes* B. Astrocytes C. Ependymal cells D. Microglia cells E. Mantle glyocytes

Microscopic analysis of a specimen revealed an organ of nervous system that consists of pseudounipolar neurons covered with glial and connective tissue membranes. Determine this organ: A Spinal ganglion* B Vegetative ganglion C Spinal cord D Cerebellum E Cortex of cerebrum

On the specimens taken from the spinal ganglion one can see neuron which has rounded body and two processes. What kind of morphological classification of neurons belong these cells? A Pseudounipolar * B Unipolar C Bipolar D Multipolar E Neuroblasts

Microscopic examination of patient revealed organ of nervous system, consisting of pseudounipolar

6 neurons which bodies are covered by glial and connective membranes. Identify present organ? A Spinal ganglion * B Vegetative ganglion C Medulla spinalis D Cerebellum E Cortex of cerebrum

A patient with a ruptured ulnar nerve was addressed in the traumatologic department of a hospital. What pathologic process occurred in the distal damaged nerve? A Development of axial cylinder in distal direction * B Degeneration of axial cylinder and myelin sheath C Development of axial cylinder in proximal direction D Degeneration of schwann cells E Thickening of myelin membrane. Sense organs. Organ of sight.

In histological preparation of the wall of an eyeball the structure which will consist of three neurons is determined. Bodies of these neurons form external, internal nuclear and ganglionar layers. What formation education of an eye has such morphological structure? A. Retina * B. Iris C. Sсlera D. Vascular shall E. Ciliar body

To the patient transplantation of the cornea is executed. What features of cornea’s structure allow to hope for her it reparation, instead of tearing away? A. Innervation B. Multilayered epithelia C. Absence of blood and typical lymphatic vessels* D. Presence of the connective tissue E. Simple epithelia

In an electronic microphoto the cell of neural origins is submitted. The terminal part of cells’ dendrite has cylindrical shape and will consist from 1000 closed membranouse disks. What it for a cell? A. Rod-shape cell * B. Neuron of spinal cord C. Neuron of spinal ganglia D. Neuron of the hemispheres E. Cones-shape cell

At the patient at inspection infringement of perception recognition of green color is revealed. What absence of cells of retina causes the given infringement of sight? A. Cones-shape cells* B. Rod-shape cells C. Pigmental epithelia D. Bipolar neurons E. Ganglionar neurons

During examining ophthalmologist found out that the patient does not distinguish blue and green, with normal perception of other colors. Dysfunction of what retinal structures is that? A Cones-shape cells * B Rod-shape cell C Bipolar neurons

7 D Amacrine neurons E Horizontal neurons

Patient’s ciliary body was damaged. What function of eye was affected? A Accommodation* B Light-conductive C Light-refracting D Protection E Trophic

The increased intraocular tension is observed in the patient with glaucoma. Secretion of aqueous humor by the ciliar body is normal. Injury of what structure of the eyeball wall caused the disorder of flow-out from the anterior chamber? A Venous sinus* B Ciliar body C Choroid D Ciliary muscle E Back epithelium of cornea

At insufficiency of vitamin A at the person twilight sight is broken. Specify cells to which poses given receptor function. A. Rod-shape cells * B. Cones-shape cells C. Bipolar neurons D. Horizontal neurons E. Ganglionar nervous cells

An infectious disease caused contractive activity of muscles that contract and dilate eye pupil (paralytic state). What functional eye system was damaged? A Accomodative * B Dioptric C Ancillary D Photosensory E Lacrimal apparatus

A histological specimen of the eyeball shows a biconvex structure connected to the ciliary body by the fibers of the Zinn's zonule and covered with a transparent capsule. Name this structure: A Lens * B Vitreous body C Ciliary body D Cornea E Sclera

A 36-year-old-patient consulted an ophthalmologist about eye ache. The examination revealed the erosion of cornea, that is the lack of superficial and spinous layers of the epithelium. What cells will provide regeneration of the damaged epithelium? A. Cells of the stratum basale* B. Cells of the stratum corneum C. Cells of the stratum granulosum D. Cells of the stratum lucidum E. Cells of the stratum superficiale

The ciliary body of a patient is damaged. Function of what eye apparatus suffers? A. Accommodation*

8 B. Light- conductive C. Photosensory D. Protective E. Trophic

A lot of people with age have clouding of the lens (phacoscotasmus or cataract) that leading to the partial or total blindness. What optical and chemical properties of the lens fibers protein will be disordered? A. Crystallin* B. Viterin C. Dinein D. Rhodopsin E. Iodopsin

In a histological specimen a structure of eyeball wall is detected; blood vessels are absent in this structure. What structure is characterized by this morphologic sign? A. Cornea* B. Ciliary body C. Choroid D. Iris E. Retina

In the microspecimen of the posterior eye wall was revealed undeveloped retinal pigment layer. What layer of the eye ball was injured during development process? A. External* B. Medial C. Internal D. Ganglion cells E. Mesenchyme

During investigation of the ocular fundus ophthalmologist drew attention at the fovea centralis of the retina. What fovea is this? A. Area of greatest visual acuity* B. Blind spot C. Place of optic nerve exit D. Place where retina’s veins exit eye E. Place without photoreceptors cells

Patient has clouding of the lens or cataract. What structure of the lens will be primary disordered in this situation? A. Lens fibers* B. Lens epithelium C. Lens core D. Lens capsule E. Ciliary fibers

In a histological specimen of an eyeball a biconvex structure is connected with the ciliary body by means of the ciliary zonulae fibers and is covered with a transparent capsule from above. Name this structure: A Crystalline lens* B Vitreous body C Ciliary body D Cornea E Sclera

9

Teenager consulted a doctor with disorder of visual acuity. Doctor explained that this problem connected with accommodation spasm. Which of mentioned components of the eye ball constitute eye accommodation apparatus? A. Ciliary muscle* B. Retina C. Vitreous humor D. Sclera E. Cornea

Condition of the macula lutea was studied during ophthalmological investigation of the ocular fundus. What retina cells elements are most found in the macula lutea? A. Cones* B. Horizontal neurons C. Pigment cells D. Rods E. Ganglion cells

As a result of punctate retinal hemorrhage a patient lost ability to see objects in the centre of visual field. In what part of retina did the hemorrhage take place? A Yellow spot* B Ciliary part of retina C Iris D Blind spot E Vascular membrane

The deficit of vitamin A causes the disorder of twilight vision. What cells is the photoreceptor function typical for? A Rod neurosensory cells* B Horizontal neurocytes C Conic neurosensory cells D Bipolar neurons E Ganglionic nerve cells

A histological specimen of an eyeball shows a structure in form of a convexoconvex formation connected with the ciliary body by the fibers of ciliary zonule and covered with a transparent capsule. Specify this structure: A Crystalline lens* B Vitreous body C Ciliary body D Cornea E Sclera

An infectious disease caused contractive activity of muscles that contract and dilate eye pupil (paralytic state). What functional eye system was damaged? A Accomodative* B Dioptric C Ancillary D Photosensory E Lacrimal apparatus

In the electron microphotograph of the sense organ revealed cells peripheral part of which constitutes from two segments. The outer segment has membrane half discs and the inner one has ellipsoid. In what organ does this structure locate?

10 A. In the organ of vision* B. In the organ of taste C. In the organ of smell D. In the vestibular organ E. In the auditory organ

In the electron microphotograph revealed cell of neural origin. Terminal part of the cell dendrite has cylindrical shape and consists from 1000 enclosed membrane discs. What cell is this? A. Cone cell* B. Ventral horn of the spinal cord C. Sensory ganglia neuron D. Cerebral cortex neuron E. Rod cell

Due to a deficiency of Vitamin A, a person has a violation in his twilight vision. Name the cells which are associated with the photoreceptor function? A Retinal rods(rod cells) * B Horizontal neurons C Bipolar neuron (cells) D Retinal cones (cone cells) E Ganglion cells

After infectious diseases the contractile activity of muscles that constricts and dilates the pupil of the eye was damaged. What functional system of the eye suffers? A Accomodative apparatus* B Dioptric apparatus C Accessory apparatus (Ancillary) D Photoreceptors E Lacrimal apparatus

A 14 year old twilight vision is damaged. The insufficiency of which vitamin gives the result? A A* B B1 C B6 D C E B12

A patient comes to an ophthalmologist after suffering an eye injury that disabled his ability to see objects in the center of the visual field. Which part of the retina was damaged? A Macula lutea (yellow spot) * B Blind spot C Iris D Ciliary body E Tunica vasculosaluvea

A 24 year old patient was admitted to a clinic with a punctured eye injury that was leaking vitreous fluid. This led to the damage of the retina. Which layer of the retina is tightly attached to the choroid and not damaged? A Pigment epithelium* B Layer of rods and cones C Ganglion cell layer D External (outer) nuclear cell layer E Inner nuclear layer

11 Due to long strains on the eyes, the ability to see distant objects deteriorates. This condition is called spasm of accommodation. How is it caused? A Contraction of cilliary body* B Disfunction of iris C Contraction of blood vessels of choroid D Trauma of cornea E Trauma of conjuctiva

In a histological slide of the eyeball, we see a layer that seperates the anterior and posterior chambers and is coloured disc with aperture in the center. What is this structure? A Iris* B Cornea C Retina D Cilliary body E Sclera

In a specimen of a fetus eyeball one observes a damaged cornea. Which part of the embryonic layer was affected during embryonic development? A Ectoderm* B Entoderm C Mesoderm D Dermatome E Nephrotome

An occular fundus was discovered in a patient who is at the ophthalmology department. Which of the following is a central fossa? A Region of the realest visual activity* B Blind spot C Place of optic nerve D Place where retinal veins exit the eye E Place without photoreceptor cells

In an experiment, an animal inflicted an injury of the cornea.Due to what willthe cornea especially its stratified epithelium regenerate? A Anterior epithelium* B Corneal stroma C The corneal endothelium D Basement membrane E Simple squamous

Nutrition of the cornea occurs by: A. Anterior chamber * B. Blood vessels propria C. Posterior chamber D. Lymphatic vessels E .Lacrimal glands

As a result of the effect of viral toxins on the cranial section of an embryo's neural tube, a child was born with an abnormal muscle tissue development. Development of which muscles was damaged? A Muscle of the iris (sphincter and dilator) * B Muscle of forearm C Muscle of hip D Muscle of stomach E Myocard

12

A histological specimen contains a part of an organ containing stratified squamous non keratinized and a connective tissue plate which consists of parallel arranged collagen fibrils. Which part of the organ is described? A Cornea* B Lens C Tunica mucosa of stomach D Tunica mucosa of mouth (oral cavity) E Retina

A neuron is presented on an electron microphotograph. It has one axon and one dendrite,in which the terminal part has a cylindrical shape and consists of 1000 closed membrane discs. What neuron is presented? A Rod neurosensory neurons* B Ventral horn of the spinal cord C Sensory ganglion neuron D Cerebral cortex neuron E Cone neurosensory neurons

Sense organs. Auditory and vestibular organs.

The animal well keeps an equilibrium antigravitational pose at inclinations of the head, at movements with linear acceleration, however frequently loses balance at movements with angular acceleration. What infringement of functions of structures is the reason of it? A. Crista* B. Macula C. Nervus vestibulocohlearis D. Lateral vestibular nucleus E. Vestibulospinal way

In a histologic preparation the receptor zone of sensoepithelial sense organ is defined. Cells of the given zone are on basal membrane and include the following kinds: external and internal sensitive, external and internal phalangeal, cells - columns, external boundary and external supporting. What sense organ posesses given receptor zone? A. Smell organ B. Taste organ C. Vestibular organ D. Auditory organ* E. Visible organ

At the patient who accepted the big dozes of streptomycin, there has come stepped loss of hearing. What function of cells of an internal ear has been damaged injured in this case? A. Deiter’s cells B. Phalanx-cells C. Cells - columns D. Sensoepithelial cells * E. Fibroblasts

A histological specimen presents a receptor zone of a sensoepithelial sense organ. Cells of this zone are placed upon the basal membrane and include the following types: external and internal receptor cells, external and internal phalangeal cell, stem cells, external limiting cells and external supporting cell. The described receptor zone belongs to the following sense organ: A Visual organ B Auditory organ *

13 C Gustatory organ D Equilibrium organ E Olfactory organ

Ampullary crests were damaged as a result of head trauma in men 32 years old. Perception of what stimuli was disrupted? A Angular acceleration* B Vibration C Gravitation D Linear acceleration E Vibration and gravitation

After having a virus disease patient has lost ability to hear. What cells of the Corti organ (spiral organ) had been damaged? A. Receptor hair cells* B. Phalangeal cells C. Outer supporting cells D. Inner supporting cells E. Pillar cells

As a result of head trauma a 32 year old man has damaged ampullas of semicircular ducts. What stimuli perception will be disturbed? A Angular acceleration* B Vibration C Gravitation D Linear acceleration E Vibration and gravitation r

A histological specimen presents a receptor zone of a sensoepithelial sense organ. Cells of this zone are placed upon the basal membrane and include the following types: external and internal receptor cells, external and internal phalangeal cell, stem cells, external limiting cells and external supporting cell. The described receptor zone belongs to the following sense organ: A Acoustic organ* B Visual organ C Gustatory organ D Equilibrium organ E Olfactory organ

During an experiment the median part of an animal's cochlea was damaged. This resulted in impaired perception of acoustic vibrations of the following frequency: A Medium* B Low C High D High and medium E Low and medium

Hearing loss is defined in a patient after an injury to the pyramid of the temporal bone. Where are the receptor cells for the bodies senses located? A Cochlea* B In the elliptical saccule C Ampulla of semicircular D In the cortex of cerebrum E -

14 What structure produces endolymph and exercises the trophic spinal organ of the inner ear? A Stria vascularis * B Vestibular(reissner) membrane C Basilar membrane D Lyphatic vessels E Perilymph

Assisting cells of the spinal organ limit space in the traingular space. What is the name of these structures? A Pillar cells* B Outer phalangeal cells and inner tunnel C Inner phalangeal cells D Supporting cells E Borders(cells of mensen)

Assisting cells of the spinal organ perform the following functions: A Support, supply, nutrition* B Afferent C Efferent D Receptor E Formation of the vestibular cochlearis

An examination of a patient revealed a voilation in the perception of vibration. the pathological process shows damage in: A Hair cells of sacculi* B Hair cells of macula of utricule C Hair cells of organ of corti D Hair cells of crystal ampullaris E Supporting cells of corti organ

Sense organs. Olfactory and taste.

In the patient with an acute rhinitis it is found out hyperemia and formation of mucus in nasal cavity is raised increased. What activity of epithelial cells of mucosa is raised increased? A. Endocrine B. Goblet cells * C. Basal D. Cells with villies E. Cilindrical

In an electron microphoto the cell of neural origins which is in structure epithelia of mucosa is submitted. Distal part of a peripheral shoot of the cell has knob-shape from which depart 10-12 cilia. What it for a cell? A. Olfactory cell * B. Bipolar neuron of spinal ganglia C. Sensitive taste cell D. Rod-shape cell E. Cones-shape cell

The boxer does not sense smell after nose trauma. Damage of what cell can lead to loss of smell? A Neurosensory cells* B Supporting epithelial cells C Basal epithelial cells D Ciliated epithelial cells

15 E Goblet cells

After prolonged inflammation of the nasal mucosa there were observed changes in the epithelium. What type of epithelium has changed? A Pseudostratified columnar* B Simple squamous C Stratified squamous D Stratified cuboidal E Stratified columnar

As a result of injury of male’s nose there was damaged mucosa covering the upper part of the superior nasal conchal. What is the result of this? A Violation of the perception of smell* B Violation air humidification C Violation of secretory activity of goblet cells D Violation warming air E Violation of warming and humidification

An electronic microphotography represents a cell of neural origin that is a component of mucous membrane epithelium. Distal part of its peripheral processes has a clavate thickening with 10-12 cilia sprouting from it. What cell is it? A. Olfactory* B. Bipolar neuron of spinal ganglion C. Sensory epithelial cells of a gustatory organ D. Rod cell of retina E. Cone cell

Skin. Skin glands. Hair and nails

End-pieces of apocrinic sweat glands contain myoepithelial cells. What function of these cells? A. Contraction * B. Secretory C. Protective D. Regeneration E. Supporting

In a histological preparation the organ which wall has the layered structure, covered keratinized epithelium is submitted. Under basal lamina there is loose connective tissue which formated papilles. Dense connective tissue forms deep layer. What organ has the given morphological attributes? A. Skin * B. Tongue C. Esophagus D. Tonsils E. Uterus

With age skin of human changed, which can be shown by reduction of its elasticity. What elements of a connective tissue most of all provide its elasticity? A. Ground substance B. Elastic fibers * C. Epidermal cells D. Collagen fibers E. Reticular fibers

In biopsy material of skin in epidermis polyhedral cells which have granules of dark brown color in

16 cytoplasm are found out. What are these cells? A. Lymphocytes B. Macrophages C. Keratinocytes D. Merkel’s cells E. Melanocytes*

The patient with hepatites complains of an itch of skin. To what function of appendages of skin this symptom is connected? A. Excretory of sweat glands* B. Protective of sebaceous glands C.Termoregulatory of sweat glands D. Protective of hair E. Protective of nails

Between epidermal cells there are many types of cell with different shape and function. Which of them are derivatives of red bone marrow? A. Langergans’ cells* B. Merkel’s cells C. Melanocytes D. Keratinocytes E. Pennet’s cells

The skin with damage of deep layer is injured. Due to what cells activity regeneration of the given layer will take place? A. Erithroblast B. Macrophages C. Lymphocytes D. Neuroblasts E. Fibroblasts*

At patient А., 12 years, on skin white stain which have no pigment. Stain has appeared after 10 years, constantly increase in sizes. What absence of skin’s cells has led to occurrence of this problem? A. Adipocytes B. Melanocytes* C. Fibroblasts D. Plasma cells E. Mast cells

In a limited area of the epidermis due to injury the layers are absent to germinativum. Name the cells that serve as the main source of its regeneration. A Basal layer cells * B Cells of strarum spinosum. C Cells of strarum granulosum. D Cells of strarum corneum. E Cells of strarum lucidum.

A patient complains of dryness of head skin, itching, fragility and loss of hair. After examination he was diagnosed with seborrhea. Disturbed activity of which cells caused this condition? A Cells of sebaceous glands * B Cells of sudoriferous glands C Epithelial cells D Adipocytes E Melanocytes

17

Study of fingerprints is used by criminalists for personal identification as well as for diagnostics of genetic abnormalities, particularly Dawn's disease. What layer of skin determines individuality of fingerprints? A Dermal papillae * B Stratum corneum epidermidis C Reticular D Stratum lucidum epidermidis E Basal

An embryo has signs of disturbed process of dorsal mesoderm segmentation and somite generation. What part of skin is most likely to have developmental abnormalities? A Derma* B Hair C Sebaceous glands D Epidermis E Sudoriferous glands

Histological study of a microslide of human skin found only dense irregular connective tissue. Which layer of this organ was analysed? A Reticular dermis* B Papillary dermis C Subcutaneous adipose tissue D Epidermis E Basal layer of epidermis

A 25-year0old-patient got an injury as result of which a portion of the nail plate was removed. What structures will be responsible for it restoration? A. Nail matrix* B. Nail fold C. Eponychium D. Subungual space E. Nail sinus

What cells in the skin epidermis together with afferent fibers terminals form tactile receptors? A. Merkel cells* B. Melanocytes C. Epidermal cells of stratum basale D. Stratum spinosum cells E. Langerhans cell

Human skin is very strong for the rupture. It is known that skin consists of epithelial tissue and two types of the connective tissue. Which from mentioned below tissues provide tensile strength of the skin? A. Dense irregular connective tissue* B. Stratified squamous epithelium C. Loose connective tissue D. Simple epithelium E. Transitional epithelium

After the skin burn stratum basale cells were suffered. What function of the epidermis will be weakening or depressed first of all? A. Regenerative* B. Protective C. Absorptive

18 D. Barrier E. Dielectric

After ultraviolet radiation over time skin darkens. Synthesis of what substance will be activated in the melanocytes after ultraviolet radiation. A. Melanin* B. Lipids C. Eleidin D. Keratin E. Kertogyalin

Under the radiation influence epidermal cells of the stratum basale were damaged. What function of the epidermis will be weakening or depressed first of all? A. Regenerative* B. Protective C. Absorptive D. Barrier E. Dielectric

The reticular layer of the skin is damaged after trauma. This layer will recover with the help of a cell population. Name this cell population. A. Fibroblast* B. Macrophages C. Lymphoblastic D. Mast cells E. Plasma cells

Cells with processes and dark brown granules in the cytoplasm were revealed in the skin epidermis of the biopsy material. What cell are these? A. Melanocytes* B. Intraepidermal macrophages C. Keratinocytes D. Merkel cells E. Lymphocytes

With age wrinkles and plications are appeared in the human skin. Changes in which skin structures mainly caused this condition. A. In the elastic fibers* B. In the collagen fibers C. In the epidermis D. In the ground substance E. In the subcutaneous adipose tissue

Histological study of a microspecimen of human skin found only dense irregular connective tissue. Which layer of this organ was analyzed? A Reticular dermis* B Papillary dermis C Subcutaneous adipose tissue D Epidermis E Basal layer of epidermis

There is histological specimen of the skin epidermis bioptic sample taken from the healthy adult. Cells division can be seen in the stratum basale. What process provide these cells. A. Physiologic regeneration*

19 B. Differentiation C. Adaptation D. Reparation E. Apoptosis

On the body human surface distinguishes areas of thick and thin skin. What skin layer has different structure in these areas? A. Epidermis* B. Papillary layer C. Reticular layer D. Derma E. Hypodermis

There are cells in the epidermis which have protective function and have mononuclear genesis. What cells are these? A. Langerhans cells* B. Melanocytes C. Keratinocytes of stratum basale D. Keratinocytes of stratum spinosum E. Keratinocytes of stratum granulosum

The study of epithelial projections of the fingerprints(dactyloscopy) for animal identification and for diagnosis of genetic anomalies, including Down syndrome. Which layer of the skin defines individual prints? A Papillary* B Corneum layer C Reticular layer D Licidum E Basale

Regeneration of the epidermis in the area of the traumatic damage occurs due to a growth zone(Malpighi's zone). Which two layers of stratified squamous epithelium are part of this zone? A Statum basale and stratum spinosus* B Stratum spinosum and stratum granulosum C Stratum granulosum and stratum lucidum D Stratum lucidum and stratum corneum E Stratum basale and stratum corneum

After a burn, a patient observes a damage to the regeneration process of the skin, following a damage to the structure located between the epithelium and connective tissue. Which structure leads to a destruction of the epithelium regeneration in this case? A Basement membrane* B Amorphous ground substens C Collagen fibres D Elastic membrane E Cell membrane

A patient was diagnosed of heat-attack as a result of long walk in rubber overalls. Which function of the skin will be damaged? A Heat production* B Blood deposition C Protective D Receptor E Exteroreceptor

20

On a micro specimen of the skin of a child's finger, we observe main components of skin -epidermis and dermis. The epidermis has properties of incomplete development. Which embryonic layer in the process of development was damaged? A Ectoderm* B Mesoderm C Entoderm D Mesenchyma E Ectomesenchyme

On an electronic microphotography of the epidermis of the skin, among the cuboidal cells are cellular processes, in the cytoplasm of which are well developed Golgi apparatus, many ribosomes and melanosomes. Name these cells? A Melanocytes* B Keratinocytes C Langerhan's cells D Merkel's cells E Epidermal cells of stratum basale Circulatory system. Heart

After physical loading arterial pressure of blood has risen increased. Why? A. Increasing quantity amount of functioning capillaries B. Increasing of maintenance contents of Kalium ions in blood plasma C. Increasing of minute volume of blood circulation* D. Increasing of maintenance contents of water in plasma E. Increasing of quantity amount of hemoglobin

As a result of the thrombosis of an artery the destruction of contractive cardiomyocytes has taken place. Due to what cells there will be reparative regeneration in a zone of damage? A. Fibroblasts * B. Cardiomyocytes C. Myosymplast D. Myosatelitocytes E. Smooth muscle cells

The serous liquid collects in pericardial cavity of the patient with pericarditis. With infringement of what cell’s activity of the pericardium this process is connected? A. Mesothelial cells* B. Endothelial cells C. Smooth myocytes D. Fibroblasts E. Macrophages

At micropreparations of heart ther were distinguished rectangular shape cells with a centrally located nucleus, developed myofibrils, linked intercalated discs. Since these cells is associated function: A Contraction of the heart* B Conduction of impulses C Endocrine D Protective E Regenerative

An organ of the cardiovascular system is at microscopic preparations. One of its layers is constructed of fibers that connected together. They are formed from cells that are connected by intercalated discs. What is the organ of the cardiovascular system?

21 A Heart* B Vein of muscular type C Artery of muscular type D Artery of elastic type E Arterioles

The man is diagnosed an epithelial tumor of the pericardium. What epithelium is a source of tumor development? A Simple squamous * B Pseudostratified epithelium C Transitional D Stratified epithelium keratinized E Stratified epithelium non-keratinized

A large amount of effusion found in the pericardial cavity of a patient with exudative pericarditis. Violation of what cell functional activity can cause this phenomenon? A *Mesotheliocyte B Fibroblasts C Contractive cardiomyocytes D Conductive cardiomyocytes E Endothelial cells

Histological examination of the human heart wall revealed the presence of endothelial, subendothelial, muscle-elastic and connective tissue layers. What is the layer of the heart wall? A *Endocardium B Myocardium C Pericardium D Epicardium E Serous

In a slide revealed organ of cardio-vascular system. One of it coats build by fibers that formed anastomosis between each other. They formed by cells which connected with the help of intercalated disks. Name this organ of cardio-vascular system. A. *Heart B. Large vein (muscle type) C. Muscle artery D. Elastic artery E. Arteriole

The organ of the cardiovascular system built from the cells interconnected by intercalated disks. What is the organ? A *Heart B Vein of muscular type C Artery of mixed type D Artery of muscular type E Aorta

The cells of dendritic shape are visible on electron microphotography of myocardium. They have few organelles but have a well-developed granular endoplasmic reticulum and secretory granules. What are these cells? A *Secretory cardiomyocytes B Ventricular cardiomyocytes C Pacemaker cells D Transitional atypical cells

22 E His' bundle cells

Large cells with light cytoplasm and eccentrically located nucleus are revealed in the histological specimen of the heart wall between endocardium and myocardium. What kind of heart cells have following morphologic signs? A. *Purkinje cells B. Pacemaker cells C. Cardiac muscle cells D. Endocrine cells E. Adipose cells

As result of left coronary artery thrombosis, the group of contractive cardiac muscle cells have been destroyed. At expense of which cells reparation in the area of damage will occur? A. *Fibroblasts B. Cardiac muscle cells C.Myosymplast D. Myosatelitocytes E.Smooth muscle cells

One of the heart wall layers similar to the blood vessel wall according to histogenesis and tissue composition. What is the source of their development? A *Mesenchyme B Splanhnotom C Endoderm D Ectoderm E Somite

Cardiomyocytes that are containing granules rich in glycoproteins found in atrial myocardium during histochemical study. What is the type of cells? A *Secretory B Contractile C P-cells D Purkinje's cell E Transitional cell

Changes in cardiomyocytes observed at microscopic examination of the fetus heart. Violation of what source led to these changes? A *Myoepicardial plate B Myotome C Endoderm D Ectoderm E Mesenchyme

The cells of rectangular shape with a centrally located nucleus, high-developed myofibrils that interconnected by intercalated discs, found at the heart microslide. What is the function of these cells? A *Heart contraction B Pulse conduction C Endocrine D Protective E Regenerative

The cells that are arranged in a light strands with a small number of myofibrils and the inclusion of glycogen found at the heart microslide. What are these cells? A *Purkinje’s fiber

23 B Pacemaker C Transitional D Endocrine E Contractile

After myocardial infarction, morphological structure of the heart wall was restored. At the expense of which tissue did regeneration occur? A*Connective B Smooth muscle C Striated muscle D Epithelial E Nervous

In a heart specimen there are detected cells with a square shape, 80-120 micrometers in size, with a centrally positioned nucleus and well-developed myofibrils connected with the help of intercalated disks. What function is related to these cells? A*Heart contraction B Nerve impulse conduction C Endocrine D Protective E Regenerative

A valvular pathology of the inner layer of the heart was revealed in a patient suffering from endocarditis. What tissue forms the heart valves? A*Dense connective tissue, endothelium B Loose connective tissue, endothelium C Cardiac muscle, endothelium D Hyaline cartilage endothelium E Elastic cartilage, endothelium

Cardiomyocytes with high-developed Golgi complex's and protein synthesis organelles are visible on the atrium electron microphotography. It is known that these cardiomyocytes produce biologically active factors. Indicate, which of these factors are produced by cardiomyocytes? A*Natriuretic factor B Insulin-like growth factor C Nerve growth factor D Calcitonin-like growth factor E Epithelial growth factor

The heart wall that is visceral to the peritoneum leaf is visible on a histological preparation. There are superficial and deep layers of collagen and elastic fibers at the connective tissue base of this layer. Which layer is this? A*Epicardium B Myocardium C Pericardium D Endocardium E Epicardium and pericardium

Arteries. Viens

At histological section of the vessel it is well expressed internal and external elastic membranes and many myocytes in the tunica media. What vessel is there in the section? A. Mixed artery B. Muscle artery *

24 C. Muscle vein D. Elastic artery E. Unmuscle vien

Walls of vessels have significant enough morphological disagreements in a structure of an average environment. Than occurrence of specific features of a structure of this environment in different vessels is predetermined? A. Maintenance contents of hormones in blood B. Influence of endocrine system C. Regulation of the central nervous system D. Inductive influence of autonomic nervous system E. Haemodinamic conditions *

Arteries of the large calibre during a systole are stretched and and come back in an initial condition during dyastole, providing stability of a blood pressure. What presence of elements of the vessel’s wall it is possible to explain it? A. Collagenic fibers B. Muscle fibers C. Reticular fibers D. Elastic fibers * E. Fibroblasts

At morphological research in section of biopsy the irregular-shaped vessel which tunica media is formed by smooth myocytes and layers of a connective tissue is determined. What is a vessel? A. Muscle vein* B. Muscle artery type C. Lymphatic vessel D. Venule E. Аrteriola

At the preparation blood vessels are presented. The intima is formed by endothelium and subendothelium, tunica media - bundles of smooth muscle cells, layers of loose connective tissue. The tunica adventitia is highly developed and formed by loose connective tissue and some smooth muscle cells. What is this vessel? A Vein of muscular type * B Artery of muscular type. C Nonmuscular vein D Artery of mixed type E Artery of elastic type

At histological section there is shown a blood vessel. Its inner coat is composed by endothelium, subendothelium and internal elastic membrane. The middle coat is enriched with smooth myocytes. Such morphological characteristics are typical for the following vessel: A Muscular-type artery * B Elastic-type artery C Capillary D Non-muscular vein E Muscular-type vein

A vessel presents at histological section. Its wall consists of endothelium, basal membrane and loose connective tissue. What type of vessel is it? A Vein of non-muscular type * B Artery C Vein of muscular type

25 D Hemocapillary E Lymphocapillary

A histological section of spleen there was identified a vessel with a wall consisting of endothelium and subendothelial layer, median coat is absent, exterior membrane knit together with connective tissue layers of spleen. What vessel is it? A Vein of non-muscular type * B Vein of muscular type C Artery of muscular type D Arteriole E Capillary

At histological section presents an artery. One of the membranes of its wall has flat cells lying on the basal membrane. What type of cells is it? A Endothelium * B Mesothelium C Smooth myocytes D Fibroblasts E Macrophages

During investigation of skin bioptat in the derma revealed vessels which have thick layer of smooth muscle cells in the tunica media. What is the name of these vessels? A. *Muscular artery B. Capillaries C. Arterioles D. Venules E. Arterio-venous shunts

In a histological specimen of tubular organ dyed with orcein have been detected about 50 thick membranes which have wiggly appearance and formed tunica media of this organ. Name this organ. A. *Aorta B. Muscular artery C. Esophagus D. Trachea E. Heart wall

In a histological specimen one can see an artery. There are flat cells on the basal membrane in one of the tunics of its wall. Name these cells. A. *Endothelium B. Mesothelium C. Smooth muscle cells D. Fibroblasts E. Macrophages

50 mm thick membranes with a wavy shape that form the middle tunic of an organ were found in the tubular organ preparation stained by orcein. What is the organ? A*Aorta B Artery of muscular type C Esophagus D Trachea E Wall of the heart

Patient suffers from the aortic aneurysm. Which of the following aortic wall structures are disordered? A *Elastic membranes

26 B Endothelium C Subendothelium D Smooth myocytes E The outer tunic

Obliterating atherosclerosis causes changes in the vessels of the lower extremities. A histological specimen of such a vessel evidently presents both internal and external elastic membranes; middle membrane contains a lot of smooth muscle cells. What vessel is affected in case of this disease? A *Artery of muscular type B Artery of elastic type C Artery of mixed type D Vein with strongly developed muscles E Lymph node

A histological specimen presents an artery. One of the membranes of its wall has flat cells lying on the basal membrane. What type of cells is it? A *Endotheliocytes B Mesotheliocytes C Smooth muscle cells D Fibroblasts E Macrophages

Wall of the vessels often have huge morphological difference in the structure of the tunica media. What is the reason of appearance such specific peculiarities in the structure of this tunica in different vessels? A. *Hemodynamic condition B. Influence of endocrine system organs C. Regulation from central nervous system D. Inductive influence of vegetative ganglion neurons E. Large content of catecholamines in the blood

Large arteries during systole stretch out and return in previous condition during diastole providing stability of bloodstream. What elements of vessel wall will explain this? A. *Elastic fibers B. Muscle fibers C. Reticular fibers D. Collagen fibers E. Large amount of fibroblasts

In the vessel slide one found prominent internal and external elastic membranes and a lot of smooth muscle cells in the tunica media. What type of vessel is it? A. *Muscular artery B. Small artery (mixed) C. Large vein (muscular) D. Elastic artery E. Extraorganic lymphatic system

Tunica intima of a vessel is lined with epithelium from within. What epithelium is this? A. *Endothelium B. Mesothelium C. Epidermis D. Transitional epithelium E. Pseudostratified epithelium

27

A specimen of pia mater includes a vessel whose wall doesn't have the tunica media, the tunica externa is adherent to the surrounding tissues, the intima is composed of a basement membrane and endothelium. What vessel is it? A Nonmuscular vein * B Muscular vein with underdeveloped muscular elements C Muscular artery D Arteriole E Artery of mixed type

The contraction of the smooth muscle cells of arterioles occurs after the adrenalin releasing from the adrenal medulla into the blood. What are the features of these vessels structure? A *Availability of perforations in the endothelium basement membrane and the internal elastic membrane B Single position of the smooth muscle cells C Presence of the effector endings on the pericytes D Presence of the contacts between the pericytes E Availability of precapillary sphincters

On a electron micrograph of a tunica intima fragment revealed cells which rests on the basal membrane and connected with each other by means of desmosomes and zonulae occludentes. Name these cells. A. *Endothelium B. Mesothelium C. Epidermis D. Epithelioreticular cells E. Macrophages

Arterioles play an important role in the blood supply of the organs functional units. Which of these structures perform this function? A *Myocytes B External elastic membrane C Internal elastic membrane D Special connective tissue cells E Endothelial

From 40 to 60 fenestrated elastic membranes were founded at the histological preparations stained by the orcein. Name this vessel. A *Artery of elastic type B Artery of muscular type C Artery of mixed type D Vein of muscular type E Vein of unmuscular type

Morphological examination revealed in histological specimen of biopsy material an irregular-shaped vessel. Its middle membrane is formed by bundles of smooth myocytes and layers of connective tissue. What type of vessel is it? A *Vein of muscular type B Artery uf muscular type C Lymphatic vessel D Venule E Arteriole

A specimen of the pia mater shows a vessel with no middle membrane in its wall, its outer membrane adheres to the surrounding tissues, the inner membrane is made up of the basal membrane and

28 endothelium. Specify this vessel: A *Fibrous vein B Muscular vein with weakly developed muscular elements C Muscular artery D Arteriola E Mixed artery

Histological specimen presents a vessel the wall of which consists of endothelium, basal membrane and loose connective tissue. What type of vessel is it? A *Vein of non-muscular type B Artery C Vein of muscular type D Blood capillary E Lymphatic capillary

A histological specimen of spleen shows a vessel with a wall consisting of endothelium and subendothelial layer, median membrane is absent, exterior membrane inosculates with the layers of spleen connective tissue. What vessel is it? A *Vein of non-muscular type B Vein of muscular type C Artery of muscular type D Arteriole E Capillary

A specimen of pia mater includes a vessel whose wall doesn't have the tunica media, the tunica externa is adherent to the surrounding tissues, the intima is composed of a basement membrane and endothelium. What vessel is it? A *Nonmuscular vein B Muscular vein with underdeveloped muscular elements C Muscular artery D Arteriole E Artery of mixed type

During morphological investigation in a histological specimen one can see irregularly shaped vessel tunica media of which formed by bundles of smooth muscle cells and layer of connective tissue. What type of vessel is it? A *Large vein (muscular type) B Muscular artery C Lymphatic vessel D Venule E Arteriole

A specimen of the pia mater shows a vessel with no middle membrane in its wall, its outer membrane adheres to the surrounding tissues, the inner membrane is made up of the basal membrane and endothelium. Specify this vessel: A *Fibrous vein B Muscular vein with weakly developed muscular elements C Muscular artery D Arteriole E Mixed artery

A histological specimen shows a blood vessel. Its inner coat is composed by endothelium and subendothelial layer. The middle coat consists of smooth muscle cells bundles. Tunica adventitia well developed and consists of loose connective tissue and some smooth muscle cells. Such morphological

29 characteristics are typical for which vessel? A *Muscular vein (large vein) B Muscular artery C Non-muscular vein D Mixed artery (small artery) E Elastic artery

Microvasculation

On electronogramm the capillary has fenestres in endothelium and continuous basal lamina. Named type of the capillary. A. Sinusoidal B. Continuous C. Fenestrated* D. Typical E. Atypical

For a capillary characteristic presence pores in endothelium and basal lamina. What is type of this capillary? A. Typical B. Continuous C. Fenestrated D. Sinusoidal * E. Lymphatic

In histological preparation there are vessels that look like flattened endothelial tubes do not contain basal lamina and pericytes, endothelium of blood vessels is fixed to the collagen fibers of connective tissue. What is this vessel? A Lymph capillary * B Haemocapillary C Arteriolar D Venules E Arteriolar-venule anastomoses

In the preparation there is one of the vessels of microvascular rate, the tunica media is formed by 1-2 layers of smooth muscle cells which are located singly and have a spiral direction. The tunica adventitia is represented by a thin layer of loose connective tissue. Specify the type of vessel. A Arterioles* B Venules. C Capillary. D Artery E Arteriolar-venule anastomoses

Intralobular capillaries of a liver specimen have wide irregular lumen. Basal membrane is absent in the major part of the capillary. What type of capillaries is it? A Sinusoid* B Visceral C Somatic D Precapillaries E Postcapillaries

On the electron micrograph of capillary are revealed fenestrae between endothelial cells and partial or total absence of basal lamina underlying the endothelium. What type of capillary is it?

30 A. Sinusoidal* B. Somatic C. Visceral D. Atypical E. Shunt

In the slide of the microvascular bed vessel tunica media consists of 1-2 layers of smooth muscle cells which have spiral orientation. Tunica adventitia is a thin, sheath of loose connective tissue. What vessel is it? A. Arteriole* B. Venule C. Capillary D. Postcapillary E. Arterio-venous shunt

In the microspecimen of red bone marrow there were revealed multiple capillaries through the walls of which mature blood cells penetrated. What type of capillaries is it? A Sinusoidal* B Fenestrated C Somatic D Visceral E Lymphatic

In histological slide are present blind ended vessels which have shape of oblate tubes. They don’t have basal membrane and pericytes. The outer surface of the endothelium is attached to the surrounding connective tissue by anchoring fibers. What type of vessel is it? A. Lymph capillary* B. Blood capillary C. Arterioles D. Venules E. Arterio-venous shunts

A small diameter vessel was detected on the electron microphotography. It is characterize by absence of basal and pericytes and presence of anchor fibrils. Which type of vessel is this? ALymphoid capillary* B Arterioles C Venules D Hemocapillary of sinusoid type E Hemocapillary visceral type

A vessel that has a structure of endothelial tube is found in the preparation of loose connective tissue. It blindly begins and characterizes by absence of basal lamina and pericytes. Endothelial cells are fixed by anchor fibrils. What is this vessel? ALymphoid capillary* B Arteroiles C Venules D Hemocapillary of sinusoidal type E Hemocapillary visceral type

Bone marrow. Haemocyto-poiesis.

After a massive irradiation of mice in laboratory conditions there was a destruction of haemopoietic cells. At what class of haemopoietic cells there is stimulation of haemopoietins? А. I class

31 В. II class * С. III class D. Blasts Е. Defferentiated cells

On a preparation the big polyhedral cells, with invagination on a surface in which are located erythroblasts is defined determined. What organ is submitted on a preparation? A. Thymus B. Tonsils C. Red bone marrow* D. Spleen E. Lymph nodes

On examination punctate of red bone marrow ther were found a significant decrease in the number of megakaryocytes. What will be with blood cells? A Decrease the number of platelets* B Decrease the number of red blood cells C Decrease the number of eosinophils D Decrease the number of neutrophils E Decrease the number of B-lymphocytes

In myeloid tissue punctate ther were found cells in which during the process of differentiation pycnosis and removing the nucleus are happened. What kind of hematopoiesis, characterized by morphological changes in the data. A Erytrocytopoiesis * B Trombocytopoiesis. C Hranulocytopoiesis D lymphocytopoiesis E Monocytopoiesis

Granulocytes are found at histological examination of biopsies of red bone marrow. What changes take place in the nucleus during differentiation of these cells? A Segmentation* B Poliployidizatsiya. C Piknoz D enucleation. E Increasing the size.

An electronic microphotograph shows a macrophagic cell with erythrocytes at different stages of differentiation located along its processes. This is the cell of the following organ: A Red bone marrow * B Thymus C Spleen D Tonsil E Lymph node

In the microspecimen of red bone marrow there were revealed multiple capillares through the walls of which mature blood cells penetrated. What type of capillares is it? A Sinusoidal* B Fenestrational C Somatical D Visceral E Lymphatic

32 In course of an experiment a big number of stem cells of red bone marrow were in some way destructed. Regeneration of which cell populations in the loose connective tissue will be inhibited? A Of macrophages * B Of fibroblasts C Of pigment cells D Of lipocytes E Of pericytes

Patient has disordered processes of erythropoiesis, granulocytopoiesis, monocytopoiesis and thrombocytopoiesis. Which hematopoietic organ had been affected? A. *Red bone marrow B. Thymus C. Spleen D. Lymph node E. Tonsil

In the slide of the human red bone marrow smear between myeloid row of cells and adipose cells one can find star shaped cells with oxyphilic cytoplasm which contacts with each other by means of their processes. Call these cells. A. *Reticular B. Fibroblasts C. Macrophages D. Dendrite cells E. Osteocytes

A specimen of the human red bone marrow smear revealed accumulation of giant cells located near sinusoidal capillaries. Call formed elements of blood which formed from these cells. A. *Platelets B. Red blood cells C. White blood cells D. Lymphocytes E. Monocytes

In the experiment in the human red bone marrow ribosomes in the polychromatophilic erythroblasts were destroyed. Which specific protein synthesis will be disordered? A. *Globin B. Fibrinogen C. Collagen D. Elastan E. Laminine

During postembryonal hematopoiesis in the red bone marrow the cells of one of the cellular differons demonstrate a gradual decrease in cytoplasm basophilia as well as an increase in oxyphilia, the nucleus is being forced out. Such morphological changes are typical for the following haemopoiesis type: A *Erythropoiesis B Lymphopoiesis C Neutrophil cytopoiesis D Eosinophil cytopoiesis E Basophil cytopoiesis

A 46 year old patient was admitted to the hematological department. It was found that he had disorder of granulocytopoesis and thrombocytogenesis processes. In what organ does this pathological process take pace? A *Red bone marrow

33 B Thymus C Spleen D Lymphatic ganglion E Palatine tonsil

In the electron microphotograph one can see macrophage along processes of which situated erythrocytes at different stages of differentiation. What organ is it? A. *Red bone marrow B. Thymus C. Spleen D. Tonsil E. Lymph node

In the slide one can see the organ stroma of which consists of reticular tissue, adipose cells, macrophages and osteogenic cells. What organ is represented in the slide? A. *Red bone marrow B. Spleen C. Thymus D. Lymph node E. Tonsil

An organ was determined in a histological slide. The parenchyma of this organ consists of haematopoietic tissue. The structure of this tissue contains erythroblasts, megakaryocytes and other cells. Identify this organ? A *Red bone marrow B Lymph node C Spleen D Thymus E Tonsils

In order of a diagnose patient a doctor needed to puncture the parenchyma of a blood forming organ. Megakaryocytes were found. What organ is this? A *Red bone marrow B Lymph node C Spleen D Thymus E Tonsils

A 46-year old patient was admitted to the immunological department. Disorders of granulopoiesis and thrombopoiesis were found. In which of the following organs do these pathological process occur? A *Red bone marrow B Thymus C Spleen D Lymph node E Palatine tonsils

Thymus. Palatine Tonsils.

At the newborn child it is revealed hypoplasia of thymus. What kind of haemopoiesis will be broken? A. Lymphopoiesis* B. Neutrophilopoiesis C. Erythropoiesis D. Monocytopoiesis E. Megakariocytopoiesis

34

On histological preparation the organ wich consists of lobules that are surrounded by layers of connective tissue. At the periphery of lobules the number of cells is more than in the center, lymph nodules are absent. Which organ is represented? A Thymus* B Red bone marrow C Lymph node D Spleen E Tonsils

The antigen independent proliferation and differentiation of T lymphocytes are studied in the child with impaired immune reactivity. Punctate of what organ was taken for study? A Thymus*. B Spleen. C Lymph nodes. D Red bone marrow. E Tonsils

Histological examination of a 40 y.o. man's thymus revealed decreased share of parenchymatous elements, increased share of adipose and loose connective tissue, and its enrichment with thymus bodies. The organ's mass was unchanged. What phenomenon is it? A Age involution* B Accidental involution C Hypotrophy D Dystrophy E Atrophy

The specimens present sections of haemopoetic and immunogenetic organs. Organ has lymph tissue forming different structures (lymph nodes, lobules, bars). In what organ does antigen-independent proliferation and differantiation take place? A Thymus* B Lymphatic nodes C Spleen D Hemolymph nodes E Tonsil

A new born baby has an underdeveloped thymus. Which type of hematopoiesis will be damaged? A *Lymphopoiesis B Monocytopoiesis C Erythropoiesis D Granulocytopoiesis E Megakariopoiesis

In the specimen one can see an organ which consists from lobules and stroma includes epitheliocytes with processes. What organ is represented in the slide? A. *Thymus B. Red bone marrow C. Spleen D. Tonsil E. Lymph node

Thymosin antibodies were injected into animals in an experiment. Differentiation of which cells will be damaged in the first place? A *T-lymhocytes

35 B Monocytes C Plasmocytes D Macrophages E B-lymhocytes

Medullary substance of a hemopoietic organ's lobule in a histological specimen is lighter coloured and contains epithelial bodies. What organ are these morphological preperties typical for? A *Thymus B Lymph node C Spleen D Liver E Kidney

Slide of hematopoietic organ is under investigation. This organ consists of differently shaped lobules. In each of the lobule presents cortex and medulla. Which organ posses these morphological characteristics? A. *Thymus B. Lymph node C. Spleen D. Tonsils E. Appendix

Under condition of experiment in the body of investigated animal was injected antibody against thymus hormones. Which cells differentiation will be affected first of all? A. *T lymphocytes B. Monocytes C. Plasma cells D. Macrophages E. B lymphocytes

During infections and intoxication in the lobules of thymus amount of epithelioreticular cells and Hassall’s corpuscles are increased and area of medulla became larger. Give name of these changes in the thymus. A. *Accidental involution B. B immunodeficiency C. Thymico-lymphatic condition D. Age involution E. T immundeficiency

During histological investigation of 40 years old man thymus was revealed decreasing of the thymus parenchyma, increasing of adipose and loose connective tissue, enriching of Hassall’s corpuscles while weight of the organ stay the same. What is the name of this phenomenon? A. *Age involution of the thymus B. Accidental involution of the thymus C. Thymus hypotrophy D. Thymus dystrophy E. Thymus atrophy

The child was born with immunodeficiency. Cell mediated immunity has been affected that has caused often viral infections. Which organ has been damaged? A. *Thymus B. Red bone marrow C. Lymph node D. Spleen E. Tonsil

36

15 years old patient during tonsillitis has enlarged his tonsils. Which histological structures of these organs take place in immune protection of the body as a response of streptococcus invasion? A. *Lymphatic nodules B. Stratified squamous keratinized epithelium C. Stratified squamous non keratinized epithelium D. Loose connective tissue E. Crypt

An organ of the oral cavity has several folds of the mucous membrane, lamina propria of which contains numerous lymphoid follicle. What is this organ? A *Tonsils B Tongue C Parotid gland D Sublingual gland E Submandibular gland

Spleen. Lymph nodes.

At morphological research of a spleen activization of immunity reactions in an organism has been determined. In what structures of the given organ begins secondary proliferation of T-lymphocytes? A. Mantile zone of white pulp B. Red pulp C. Periarterial zone of white pulp* D. Marginal zone of white pulp E. Central zone of white pulp

On preparation the organ is submitted, in reticular stroma which matures cells of blood and lymph nodules are placed. What organ is submitted on preparation? A. Tonsils B. Тhymus C. Spleen * D. Lymph node E. Red bone marrow

At the preparation the organ is represented wich covered connective tissue capsule from which trabeculae away. In the organ you can distinguish cortex, which contains lymphatic nodules, and the medulla with lymphoid cells. Which organ is represented on the preparation? A Lymph node* B Thymus C Spleen D Red bone marrow E Tonsils

At histological preparation the organ’s parenchyma is represented by lymphoid tissue that forms the lymph nodules, the latter are diffusely and contain a central artery. Which organ is represented on the preparation? A Spleen* B Tonsils C Lymph node D Thymus E Red bone marrow

At histological preparation there is an organs which cells form three types of lymphatic structure:

37 lymphatic nodules, sinuses and medullary cord. Which organ is represented? A Lymph node* B Spleen C Thymus D Tonsils E Red bone marrow

The spherical formations of lymphocytes were found on micropreparations. In the middle of formations - the central artery. Which organ is examined? A Spleen. * B Kidney. C Thymus. D Bone marrow. E Lymph node.

A student examinated two histological preparations. Both of them had organs that had lymph nodules. On the first preparation - only follicles, and on the second- follicles contained eccentrically vessel. Determine what kind of organ? A First - lymph node, the second - spleen* B The first- red bone marrow, the second-spleen C The first - thymus, the second - spleen D The first -liver, the second-lymph node E The first - liver, the second-spleen

In a histological specimen parenchyma of an organ is represented by lymphoid tissue that forms lymph nodes; the latter are arranged in a diffuse manner and enclose a central artery. What anatomic formation has such morphological structure? A Spleen * B Tonsil C Lymph node D Thymus E Red bone marrow

A specimen shows an organ covered with the connective tissue capsule with trabeculae radiating inward the organ. There is also cortex containing some lymph nodules, and medullary cords made of lymphoid cells. What organ is under study? A Lymph node * B Thymus C Spleen D Red bone marrow E Tonsils

In the specimen one can see an organ where lymphocytes formed 3 types of lymphoid structures such as lymphatic nodules, medullary cords and sinuses. What organ is it? A. *Lymph node B. Spleen C. Thymus D. Tonsil E. Red bone marrow

A histological specimen presents an organ that has both cortical and medullary substance. Cortical substance consists of an external zone that contains lymph nodules as well as of a paracortical zone. Medullary substance contains medullary cords, sinuses and trabeculae. What organ possesses these morphological signs?

38 A *Lymph node B Spleen C Kidney D Thymus E Adrenal glands

In the slide which was made from the spleen one can see white and red pulp in the base of which is rest special tissue which formed their stroma. What tissue is it? A. *Reticular connective tissue B. Dense connective tissue C. Adipose tissue D. Muscle tissue E. Nervous tissue

In a histological specimen parenchyma of an organ is represented by lymphoid tissue that forms lymph nodes; the latter are arranged in a diffuse manner and enclose a central artery. What anatomic formation has such morphological structure? A *Spleen B Tonsil C Lymph node D Thymus E Red bone marrow

In the histological cross section of the lymph node in the experimental animal after antigen stimulation in the medullary cords one can find huge amount of cells with intensively basophilic cytoplasm, eccentrically positioned nucleus with chromatin giving the illusion of the spokes of the wheel and light area of cytoplasm near it. Call these cells. A. *Plasma cells B. Macrophages C. Fibroblasts D. Adipose cells E. Mast cells

Student got 2 histological specimens. They both have lymphatic nodules. First slide has only follicles but second one has follicles with eccentrically positioned vessel. Determine these slides. A. *First-lymph node, second-spleen B. First-red bone marrow, second -spleen C. First thymus, second- spleen D. First liver, second -lymph node E. First-liver, second -spleen

In the biopsy sample of the lymph node were revealed focuses of increased formation of plasma cells. Antigen depended stimulation of which immune cells have caused their formation? A. *B lymphocytes B. T lymphocytes C. Macrophages D. Dendrite cells E. Interdigital cells

Examination of a patient who was exposed to the ionizing radiation revealed damage of white pulp. What cells of white pulp undergo pathological changes? A *Lymphocytes B Neutrophilic leukocytes C Basophilic leukocytes

39 D Monocytes E Tissue basophils

Morphological investigation of the spleen revealed activation of immune reactions in the organism. In which structures of this organ do antigen depended proliferation of T lymphocytes begin? A. *Periarterial sheath of white pulp B. Central zone of white pulp C. Germinal center D. Marginal zone of white pulp E. Red pulp

In the specimen was revealed an organ in the reticular stroma of which situated blood formed elements and seen lymphoid formation. What organ is this? A. *Spleen B. Lymph node C. Tonsil D. Thymus E. Red bone marrow

In a specimen was revealed roundish formation of the lymphocytes with a central artery in the center. What organ is it? A *Spleen B kidney C Lymph node D Thymus E Red bone marrow

One has done histological section through lymph node. In the slide one can see enlargement of it paracortex. Proliferation of what cells of lymph node have caused this process? A. *T lymphocytes B. Dendritic cells C. Plasma cells D. Macrophages E. Reticular cells

In a microscopic specimen is a bean-shaped organ which has cortical and medullar substance. Cortical substance is represented by separate spherical nodules 0,5-1 mm in diameter, medullar substance – by medullary cords. What organ is this? A. *Lymph node B. Kidney C. Thymus D. Adrenal gland E. Spleen

Damage to the white pulp of the spleen is found in an examination of a patient who has undergone ionizing radiation. Which cells of the spleen were damaged with these pathological changes? A *Lymphocytes B Neutrphilic leucocytes C Basophilic leucocytes D Monocytes E Tissue basophils

A cortex and medulla are present in a histological preparation of an organ. Outer of the cortex consists of lymph nodules. The medulla contains bands, sinuses and trabeculae. What is this organ?

40 A *Lymph node B Spleen C Kidney D Thymus E Adrenal

System of the immunity defence.

At older people’s frequency of occurrence of tumors rises. One of principal causes of it: A. Decreasing of intensity of antibodies’ formation B. Increasing of frequency of infringements mitosis C. Decreasing of activity of cellular immunity* D. Increasing of activity of cellular immunity E. Increasing of activity of antibodies’ formation

To the patient with the big burns have made change of a donor’s skin. But for 8 day transplantant’s color has changed and for 11 day it started to be torn away. What cells accepted in it participation? А. T-lymphocytes* B. Erythrocytes С. B-lymphocytes D. Acidophills E. Basophills

A patient 30 years was diagnosed with skin tumor. What epidermal cells take part in the immune response? A T-lymphocytes* B Keratinocytes C Keratinocytes and Merkel cells D Merkel cells E Cells of stratum spinosum

Burn wound was closed pig skin (heterotransplantation). Name the effector cells with reject the transplant (pig skin). A T-killer* B T-helper C T-suppressor D B lymphocytes E Natural killer

Antibodies are produced when antigen enters the body repeatedly. With the function of what immune cells this phenomenon linked? A Memory lymphocytes* B T - killer C T - suppressor D Macrophages E Dendritic cells

In girl’s blood 16 years old which suffers autoimmune inflammation of thyroid gland, numerous plasma cells are found out. With proliferation and differentiation of what blood cells connects increase in quantity amount plasma cells? A. T-lymphocytes B. T-helpers C. Mast cells

41 D. Т-killers E. B-lymphocytes *

To prevent epidemic of California virus vaccine (heterogenous protein) was injected in the organism of human. What cells will take place in specific immunity? A. *Lymphocytes B. Adipose cells C. Pigmentocytes D. Fibroblasts E. Adventitial cells

A patient with clinical presentations of primary immunodeficiency displays disturbance of antigen- presenting function by immunocompetent cells. What cells may have structure defect? A *Macrophages, monocytes B T-lymphocytes C B-lymphocytes D Fibroblasts E 0-lymphocytes

Plasma cell produces specific antibody for specific antigen. During injection of antigen quantity of plasma cell increased. At the expense of what blood cells occurred increasing of plasma cells quantity? A. *B lymphocytes B. Eosinophils C. Basophils D. T lymphocytes E. Monocytes

Rejection of transplant developed in the patient after transplantation of heterogenous kidney. Call main effector cells, which take place in this immune reaction? A. *T killer B. B lymphocytes C. T suppressor D. T helper E. Plasma cells

During heterotransplantation of the organ was revealed rejection of transplant. What blood cells will ensure this process? A. *T killer B. T helper C. T suppressor D. T 0 lymphocytes E. T memory cells

Acquired Immune Defficiency syndrome was diagnosed after researching a patients blood. The decrease of which cells will be experienced in the first place? A *T-helper B T suppressor C T killer D T memory E Lymphocytes

During the development of the immune response, as a response to pathogenic agents, T-cells differentiate into antigen reactive killer, helper and suppressor cells. This doesn't occur in which of the following organs?

42 A *Thymus B Spleen C Lymph node D Tonsils E Gastric lymph nodes

Endocrine system. Hypothalamus. Epiphysis.

Woman 40 years old have powerless childbirth, caused by weakness of contractile abilities of myometrium. What hormonal preparation needs to be entered to help her? A. Аldosterone B. Cortisol C. Dexamethasone D. Оxitocine * E. Prednisolone

Patient K., 35 years old complains about permanent thirst, bad appetite. He drinks every day 9 L of fluid. Daily diuresis increased, urine discolored. Most probable reason of such pathology development in this patient is: A. *Hypothalamic nucleuses B. Epithelia of nephron tubules C. Adenohypophysis D. Pineal gland E. Basal membrane of glomerulus capillaries

Cessation of bleeding after parturition connects with action of oxytocin to the uterus wall. Which layer of the organ does react at the action of this hormone? A. *Myometrium B. Endometrium C. Perimetrium D. Parametrium E. Submucous

40 years old woman has weak labor activity caused by weak contraction of myometrium. What hormone should be injected to help this woman? A. *Oxytocin B. Hydrocortisone C. Dexametasone D. Aldosterone E. Prednisolone

Experimental animal produces big amount of urine and have strong thirst. Urine doesn’t have sugar. What cells disordered? A. *Neurosecretory cells of supraoptic nucleus B. Follicular endocrine cells of the thyroid gland C. Principal cells D. Endocrine cells of the zona glomerulosa of adrenal gland E. Endocrine cells of the medullary region of adrenal gland

Polyuria developed in a patient with bleeding in the anterior part of the hypothalamus. The defficiency of which hormone influences the reabsorption of water in the kidney tubules? A *Vasopresin B Oxytocin C Adrenalin

43 D Calcitonin E Aldesteron

In the slide endocrine system organ is represented. It surrounded by connective tissue capsule which extends trabeculae in the center of the organ and formed lobules. Each lobule contains two types of cells Neurosecretory pinealocytes – polygonal cells with processes located in the center and glial cells (astrocytes) located in the periphery. What organ is this? A. *Pineal gland B. Hypophysis C. Hypothalamus D. Thyroid gland E. Medullary region of the adrenal gland

Hypophysis. Hypothalamo-hypophyseal system.

The patient of very high growth comes to the doctor. He has long thick fingers of hands, the big bottom jaw and the drooped lower lip. Increased secretion of what hormone can be suspected? A. Somatotrophin* B. Thyrotroph C. Gonadotroph D. Vasopressin E. Oxitocin

Growth of the child of 10 years reaches achieves 178 sm, weight - 64 kg. What endocrine gland it is connected to infringement of activity? A. Reproductive B. Suprarenal C. Thyroid D. Hypophysis* E. Parathyroid

A month later after childbirth the woman of 25 years addressed to the doctor with complaints to decrease reduction in quantity amount of milk. What reduction of secretion of hormone has led to such condition? A. Thyrotrophin B. Insuline C. Prolactine* D. Gonadotrophin E. Somatotrophin

Student investigated endocrine gland. Its parenchyma consists of epithelium and neural tissue. Epithelial trabecules have two types of cells: chromophilic and chromophobic. Identify this organ: A Hypophysis * B Adrenal glands C Hypothalamus D Thyroid gland E Parathyroid gland

Examination of the patient revealed enlargement of some body parts (jaw, nose, ears, feet, hands), but body proportions were conserved. It might be caused by intensified secretion of the following hormone: A. *Somatotropin B. Somatostatin C. Tetraiodothyronine D. Triiodothyronine

44 E. Cortisol

After sepsis 27 years old patient has bronze color of the skin which typical for Addison disease. Mechanism of hyper pigmentation based in increasing of hormone secretion. A. *Melanostimulating B. Somatotropic C. Gonadotropic D. B lipotropic E. thyreotropic

Patient has suffered from hypothyroidism for 7 years. Deficiency of thyrotropic hormones was revealed. What cells of adenohypophysis will be changed? A. *Thyrotropes B. Gonadotropes C. Corticotropes D. Somatotropes E. Mammotropes

On the background of deficiency of sex hormones in 30 years old female was revealed increased amount of follicle stimulating hormone. What cells synthesized this hormone? A. *Gonadotropes B. Thyrotropes C. Corticotropes D. Somatotropes E. Mammotropes

In the specimen of adenohypophysis between endocrine cells one can see cells cytoplasm of which is stained oxyphilly. These cells secrete prolactin. Call these cells. A. *Mammotropes B. Thyrotropes C. Adrenocorticotropes D. Gonadotropes E. Pituicytes

For morphological investigation endocrine gland was represented. Parenchyma of this gland consists of epithelium and nervous tissue. In epithelial trabeculae revealed two types of cells chromophiles and chromophobes. Name this organ. A. *Hypophysis B. Adrenal gland C. Hypothalamus D. Thyroid gland E. Parathyroid gland

50 years old patient complains about enlargement of ears, nose and hands size. Hyperfunction of which gland does these symptoms give? A. *Hypophysis B. Thyroid gland C. Sex glands D. Adrenal glands E. Pineal gland

The aim of the morphological study was to investigate an endocrine gland with parenchyma consisting of epithelium and neural tissue. In the epithelial trabeculae the study revealed two types of cells: chromophile and chromophobe. Identify this organ:

45 A *Pituitary gland B Adrenal gland C Hypothalamus D Thyroid gland E Parathyroid gland

A 32-year-old patient consulted a doctor about the absence of lactation after parturition. Such disorder might be explained by the deficit of the following hormone: A *Prolactin B Somatotropin C Vasopressin D Thyrocalcitonin E Glucagon

Examination of a 32 year old patient revealed disproportional skeleton size, enlargement of superciliary arches, nose, lips, tongue, jaw bones, feet. What gland's function was disturbed? A *Hypophysis B Epiphysis C Pancreas D Thyroid E Suprarenal

During X ray examination of the bones of the base of the cranium were revealed increasing of the cavity of sella turcica and thinning of processus clinoideus anterior and damaging of different areas of sella turcica. Tumor of what endocrine gland can cause such damaging of bones? A. *Hypophysis B. Thymus C. Pineal gland D. Thyroid gland E. Adrenal gland

The pituitary development occurs from the 2 embryonic leaflets: epithelial and neural. What is the source of development of epithelial leaflets? A*Epithelium of stomodeum B Epithelium of proctodeum C Epithelium of primary intestine D Epithelium of coelomic cavity E Epithelium of nervus tube

Disturbances of the ovary-menstrual cycle are found in a 22-year old girl. Secretion of which hormone of the hypophysis is damaged, causing these disturbances? A *Lutotropin B Folitropin C Prolactin D Somatotropic hormone E Adenocorticotropic hormone

Gigantism is observed in a person. The function of which gland is damaged? A *Hypophisis B Adrenal glands C Thyroid D Thymus E Epiphysis

46 In a woman who breast feeds a baby, the discharge of milk was decreased, but the process of secretion in lactocytes was not disturbed. The defficiency of which lead to it? A *Prolactin B Folitropin C Lutotropin D Progesteron E Oxytocin

Thyroid gland. Parathyroid glands.

The woman of 53 years, growth of 163 sm, weight of 93 kg, fat constitution, the face with edema, inactive, apathetic. Infringement of functions what endocrine gland causes a condition of the patient? A. Suprarenal B. Gonades C. Parathyroid D. Thyroid* E. Hypophysis

On histopreparation it is submitted parenchymal organ. Parenchyma consists of the many follicles which wall is formed by a layer of cuboidal cells. In a gleam of follicles collects colloid. To what organ such morphological attributes are characteristic? A. Thyroid gland* B. Ovaries C. Lymph node D. Hypophysis E. Suprarenal gland

At the patient for the third day after total thyroidectomia muscle spasmes have appeared. What medical products should be appointed this patient? A. Preparates of calcium* B. Antispasmatic drags C. Sedative drags D. Antibiotics E. Preparates of potassium

Kidneys of a man under examination show increased resorbtion of calcium ions and decreased resorbtion of phosphate ions. What hormone causes this phenomenon? A Parathormone * B Thyrocalcitonin C Hormonal form D 3 D Aldosterone E Vasopressin

Young woman comes to an endocrinologist with complains about sleepiness, depression, fatigue, bad appetite and increasing of the body weight. Disorder of what endocrine gland can cause such problems? A. *Thyroid B. Pancreas C. Adrenal cortex D. Adrenal medulla E. Ovary

Parodontitis is treated with calcium preparations and a hormone that stimulates mineralization and inhibits tissue resorption. What hormone is it? A *Calcitonin

47 B Parathormone C Adrenalin D Aldosterone E Thyroxine

A child has disturbed enamel and dentine formation as a result of decreased content of calcium ions in his blood. What hormone deficiency may cause such changes? A *Thyreocalcitonin B Somatotropin C Thyroxin D Parathormone E Triiodothyronine

A patient has the sudden decrease of Са2+ content in blood. What hormone secretion will increase? A *Parathormone B Thyrocalcitonin C Aldosterone D Vasopressin E Somatotropin

A child has abnormal formation of and dentin as a result of low concentration of calcium ions in blood. Such abnormalities might be caused by deficiency of the following hormone: A *Parathormone B Thyrocalcitonin C Thyroxin D Somatotropic hormone E Triiodothyronine

Microscopic study of an endocrine gland revealed that its parenchyma consisted of follicular structures. Their wall was formed by monolayer cubic epithelium, and their cavity was filled up with oxyphilic substance. What hormone is secreted by this gland? A *Thyroxin B Aldosterone C Cortisol D Parathormone E Oxytocin

Clinical examination of a female patient revealed reduction of basal metabolism by 40%, gain in body mass, drop of body temperature, face puffiness, sexual dysfunctions, inertness and apathy, lowered intelligence. These symptoms are caused by dysfunction of the following endocrine gland: A *Hypofunction of thyroid gland B Hypofunction of parathyroid glands C Hypophysis hyperfunction D Epiphysis hypofunction E Hyperfunction of thyroid gland

After a surgical procedure an experimental animal died from intense convulsions. What endocrinal glands were extracted? A *Parathyroid B Thyroid C Adrenal D Ovaries E Testicles

48 Examination of a patient who complains about deglutitive problem revealed a tumor-like eminence 1-2 cm in diameter on the tongue root in the region of the cecal foramen. These are overgrown remnants of the following gland: A *Thyroid B Parathyroid C Adenohypophysis D Thymus E Sublingual

Parodontitis is treated with calcium preparations and a hormone that stimulates tooth mineralization and inhibits tissue resorption. What hormone is it? A *Calcitonin B Parathormone C Adrenalin D Aldosterone E Thyroxin

Under action of harmful factors of environment in follicular cells lysosomes formation disordered. What part of hormone production in the thyroid gland will be disordered? A. *Proteolysis of phagocytized colloid from follicles B. Colloid synthesis C. Iodination of colloid D. Resorption of colloid E. Thyroglobulin synthesis

40 years old patient came to a doctor with complains about tachycardia, exophthalmos, fatiguability, reduced weight of the body. Increasing of what cells functions does this condition connect? A. *Follicular cells B. Parafollicular cells C. Parathyroid cells D. Apud cells E. Acidophil endocrine cells

30 years old patient was diagnosed with thyroid gland hyperfunction. What shape do follicular cells have in the follicles? A. *Columnar B. Polygonal C. Squamous D. Spindle E. Cuboidal

42 years old patient after resection of thyroid gland had convulsions. After injection of calcium preparation she got relief. Disorder of what endocrine glands have caused this condition? A. *Parathyroid B. Adrenal C. Ovary D. Hypophysis E. Pineal

Suprarenal gland.

At the man of 42 years which long time was in a condition of stress, in urine the contents of 17- ketosteroids is considerably increased, that first of all testifies to increase of secretion:

49 A. Epinephrine * B. Estradiol C. Kortizol D. Norepinephrine E. Aldosterone

Organ’s parenchyma forms dense glomerulus, formed from endocrinocytes. To what organ the given structure is characteristic? A. Lymph node B. Spleen C. Kidney D. Suprarenal gland* E. Pancreas

A patient has been given high doses of hydrocortisone for a long time. This caused atrophy of one of the adrenal cortex zones. Which zone is it? A Fascial * B Glomerular C Reticular D Glomerular and reticular E -

A preparation of endocrine gland demonstrates cortical and medullary substances divided with connective tissue layer. In the cortical substance parenchyma cells make up three zones: they form rounded clusters in the superficial zone, parallel strands in the middle one, and in the deep zone cells strands form net-like structure. What gland is it? A. *Adrenal B. Thyroid C. Hypophysis D. Epiphysis E. Hypothalamus

Characterizing stress students made an inaccuracy by telling that synthesis of cortical region of adrenal gland glucocorticoids is stimulated by hypophysis hormones. What clarification should he make? A. *Adrenocorticotropic hormone B. Somatotropin C. Gonadotropic hormone D. Mammotropic hormone E. Thyrotropic hormone

Microscopic examination of a parenchymatous organ revealed that its epithelial cords formed glomerular, fascicular and reticular zones. The central part of the organ was presented by accumulations of chromaffin cells. Specify this organ: A *Adrenal gland B Thyroid gland C Epiphysis D Liver E Hypophysis

In a histological specimen of adrenal cortex there are petite polygonal cells that form roundish clusters and contain some lipid inclusions. What part of adrenal is presented in this histological specimen? A *Glomerulosa zone B Intermedial zone C Fasciculata zone

50 D Reticularis zone E -

In the specimen parenchymal organ is represented. External layer of the cortex of it is formed by glomeruli created by endocrine cells. What organ is it? A. *Adrenal gland B. Lymph node C. Spleen D. Thyroid gland E. Ovary

It is known that aldosterone regulates amount of sodium in the body. What cells of the adrenal gland do synthesize this hormone? A. *Cells of zona glomerulosa B. Chromaffin cells producing epinephrine C. Cells of zona reticularis D. Cells of zona fasciculata E. Chromaffin cells producing nor epinephrine

Alimentary system. Structure of organs of oral cavity

A histological specimen of an oral cavity organ demonstrates that the organ's anterior surface is lined with multilayer squamous nonkeratinized epithelium, and its posterior surface - with ciliated epithelium. What organ is it? A Soft palate* B Gingiva C Hard palate D Lip E Cheek

In histological slide were identify the part of the oral cavity, which is based on bone tissue. It is covered by mucous membrane, which shows stratified squamous epithelium. In the formation distinguish fat, glandular and marginal zone. In all areas of the lamina propria of mucosa collagen fibers form a powerful beam that intertwines in the periosteum. What kind of structure is represented in the sample? A * Hard palate B Gums C Lips D Cheek E Tongue

One of the structure of oral cavity has a few folds of mucous membrane in the lamina propria with numerous lymphoid follicles. What kind of organ is it? A *Tonsils B Tongue C parotid gland D Sublingual gland E submandibular gland

The patient initiated a sense of taste. The overall sensitivity persists. Which papillae of the tongue are not damaged? A * Filiform B Circumvallate C Fungiform D Foliate

51 E All

A 53 years old patient complaining with the deterioration of taste sensitivity. During the examination doctor noticed the phenomenon of mucosal atrophy of certain areas of the mouth. Where are the most likely observed morphological changes? A * On the upper surface of the tongue B On the lower surface of the tongue C At the root of the tongue D On the hard palate E On the gums

During an injury one of the areas of the mouth was damaged, it has maxillary, intermediate and mandibular area. Which organ was damaged? A * Cheek B Tongue C Lips D Hard palate E Soft palate

In the biopsy of oral mucosa revealed morphological signs of the gums. What are the structural features of the mucous membrane of the gums can be observed in normal? A * Quiescent adherent to the periosteum, lamina propria forming high papillae, absent of muscular plate B Chubby adherent to the periosteum, well defined muscular plate C No muscular plate, submucosa is well-developed D No muscular plate and lamina propria E Contains many small salivary glands

In newborn baby was found median cleft lip and upper jaw. Abnormalities of which processes is caused this defect? A * Nonunion of the medial nasal processes. B Nonunion of the medial nasal processes of the maxilla. C Nonunion of the lateral nasal processes of the maxilla. D Cleft palatine processes. E Cleft maxillary processes.

During the study of histological slides of oral mucosa was found that the stratified squamous epithelium infiltrated by lymphocytes. What is the area of the mouth is most likely represented in the sample? A * Mucous membrane of the tonsils B Mucous membrane of the lips C Mucous membrane of the cheek D Mucous membrane of the hard palate E Mucous membrane of the gums

In the study of histological preparations of the mouth shows that it has in its structure three parts: skin, mucous and intermediate parts. Basis of structure forms striated muscle. What is the structure of oral cavity? A * Lips B Hard palate C Soft palate D Tongue E Gums

Dentist during examination of the oral cavity of the patient noticed that his tongue is covered with a

52 whitish bloom. What histological structures involved in its formation? A * Epithelium of filiform papillae B Epithelium of foliate papillae C Epithelium of circumvallate papillae D Epithelium of fungiform papillae E Lingual tonsils

The baby receives breast milk. What are the histological structure of oral cavity adapted for breast nipple stimulation causing reflex of milk? A * Epithelial villi of the lips B Keratinizing stratified squamous epithelium of the lips C Connective tissue papillae of the lips D Mushroom-shaped papillae of the tongue E Foliate papillae of the tongue

In histological slides was defined structure of the oral cavity, represented by mucous membrane, which is free of attached and which is firmly adherent to the periosteum. Epithelium - stratified squamous keratinized. Lamina propria form long papillae deeply resort in the epithelium. What kind of structure is it? A *Gums B Hard palate C Lip D Cheek E Tongue

In the newborn were found a defect in the sagittal line of the soft palate. Which process is displayed? A * Swallowing B Chewing C Digestion D Breathing E articulation

In histological slide identified crypts of the tonsils, the epithelium is infiltrated by leukocytes. What kind of the epithelium is a part of this structure? A *Stratified squamous nonkeratinized. B Simple columnar. C Stratified cuboidal. D Stratified squamous keratinized. E Ciliated.

The patient, 40 years old, suffering from heart attacks. A doctor appointed him receiving nitroglycerine under the tongue. What are the structural features of the mucous membrane of the mouth primarily determine the opportunity of medication? A *Permeability of the stratified squamous nonkeratinized epithelium B Permeability of the stratified squamous keratinized epithelium C Permeability of the stratified squamous epithelium D The presence of papillae of the tongue E The presence of salivary glands

The patient, 30 years old, came to the doctor with complaints of fever up to thirty eight degrees, weakness, pain in the throat. The examination revealed that the patient's tongue is covered with white bloom. What are the histological structure of the tongue involved in the formation of plaque? A * Filiform papillae B Foliate papillae

53 C Fungiform papillae D Circumvalate papillae E Connective tissue papillae of the tongue

There is a specimen of soft palate where both oral and nasal surfaces can be seen. It was revealed that oral cavity had damaged epithelium. What epithelium is damaged? A *Multistratal squamous nonkeratinizing B Multistratal cubical nonkeratinizing C Multistratal prismatic nonkeratinizing D Multistratal squamous keratinizing E Multirowed ciliated epithelium

A child damaged the lateral surface of his tongue. What lingual papillas are most likely to be damged? A *Foliate B Conic C Vallate D Filiform E Fungiform

A histological specimen of an oral cavity organ demonstrates that the organ's anterior surface is lined with multilayer squamous nonkeratinous epithelium, and its posterior surface - with multiserial ciliated epithelium. What organ is it? A *Soft palate B Gingiva C Hard palate D Lip E Cheek

In the histological slide the head end of the embryo 5 weeks of determined gill arch. Indicate what develops from the first pair of data structures? A * Mandibular and maxillary processes. B Mandibular processes. C Maxillary processes. D The external auditory canal. E Thyroid cartilage.

Excalation of which parts of the facial skull in the embryonic period leads to malformations such as "cleft palate"? A * Palatine processes B Frontal processes C Frontal and maxillary processes D Mandibular processes E Mandibular and palatine processes

Mucosa specimens are made from a child's cheeks in which we distinguish the maxilla, mandibular and intermediate zones. According to what main feature can we distinguish the intermediate zone? A. *Absence of salivary glands B. Absence of sweat glands C. Absence of epithelium tissue D. Absence of lamina propria E. Absence of microvilli

The dentist discoverd a widened space in the gingival pocket due to the detachment of the epithelium from the surface of the tooth. Which type of epithelium was damaged?

54 A. *Stratified squamous non keratinized B. Stratified flat keratinized C. Stratified cuboidal nonkeratinized D. Stratified columnar nonkeratinized E. Pseudostratified columnar epithelium

In a specimen of the tongue we distinguish the stratified squamous non keratinized epithelium, a well developed lamina propria of the mucous layer and a submucosa through which the excretory ducts of the salivary glands pass. Which part of the tongue are we observing? A. *Inferior B. Superior C. Lateralis D. Taste E. Tonsillaris

In a specimen of a tongue we observe a simple alveolar-tubular gland that is rich in mucous and has signs of interrupted secretion. Which area of the tongue with these ducts is damaged? A. *Radix of tongue B. Corpus of the tongue C. Superior surface around circumvallate papillae D. Apex of tongue E. Inferior surface

In the taste buds of the fungiform papillae of the tongue there was a loss of basal cells. What is the result? A. *Physiological preparation of the receptor and supporting cells stops B. Close taste pores of the buds C. Receptor cells taste buds lost D. Disorder of the buds innervation E. -

Term of the epithelium renewal of the hard palate mucosa takes: A. *10-12 days B. 47-57 days C. 20-25 days D. 20-90 days E. 4-10 days

The taste buds of the tongue are elliptically shaped and occupy the entire thickness of the epithelium. they contain several types of cells. As a result of what cells does the regeneration of supporting and sensory cells of the taste buds occur? A. *Basal cells intercalated cells B. Myoepithelial cells C. Receptor cells D. Taste cells E. Secretory cells

Which papillae of babies tongue are able to irritate the mothers nipple, contributing to the process of breastfeeding? A. *Conical shape of tongue B. Foliate papillae C. Circumvallate D. Fungiform E. Filiform

55

Partial absence of papillae is detected in a patient with diamond shaped glossitis. What papillae are located on the sides of an adults tongue? A. *Foliate B. Filiform C. Conical shape D. Fungiform E. Circumvallate

A 3 year old has lost the sense of taste due to thermal burns on the lateral surface of the tongue. Which cells will be the source of functional recovery of these taste buds? A. *Basal cells B. Supporting cells C. Sensory cells D. Ito cells E. Epithelial cells

During the examination of the mucous membrane of the hard palate a dentist discovered a rounded formation located in the lamina propria of the mucous in the seam. Clusters of which cells were formed? A. *Epithelial cells B. Lymphocytes C. Adipocytes D. Pigment cells E. Mucous cells

Inadequate removal of the edges of enamel from the second small molar tooth led to its breaking and the formation of sharp edges that injure the mucosa of the cheek. What structures were damaged? A. *Epithelial cells and the lamina propria B. Epithelial cells and aponeurosis C. Epithelial cells and bone D. Epithelial cells and glands E. Epithelial cells and muscles

While filling a cavity of the first class without a previously fixed matrix the filling material entered the interdental space and injured the interdental papillae. Which structures were damaged? A. *Epithelial cells and lamina propria B. Epithelial cells and submucosa C. Epithelial cells and bone D. Epithelial cells and glands E. Epithelial cells and muscles

During the trepanning of the ruminants surface of an intact crown of a tooth, the boron slipped and injured the soft tissue of the gums. Which tissue has been damaged? A. *Epithelial cells and lamina propria B. Epithelial cells and submucosa C. Epithelial cells and pulp D. Epithelial cells and glands E. Epithelial cells and muscles

Tooth structure.

Rounded structure formations were detected in the coronal part of the pulp in a 42-year old patient suffering from paradontosis. Name these structures?

56 A. *Denticles B. interglobular dentine C. Sclerotic dentine D. Dead dentine E. Dental stones Examination of a tooth slice of a 42 y.o. man revealed on the dentinal-enamel border some solid linear fusiform structures as long as 1/3 of enamel depth. What structures were revealed? A *Enamel spindles B Denticles C Enamel fascicles D "Dead" tracts E Carious damage

A histological preparation of multyrooted tooth reveals polygonal cells with processes in the root bifurcation area. What cells and what dental tissues are characterized by this morphologic features? A. * Cementocytes, B. , enamel C. Enameloblasts, enamel D. Fibroblasts, pulp E. Cementocytes, dentine

In histological slide of mandibular tooth germ of embryo turns in which the formed by small colored basophilic stellate cells. What tissue forms part of these dental germs? A * Mesenchyme B. Epithelial C Reticular D Cartilage E Bone

On electronic microscopy of periodontal tissue fibers were found that one of their ends dipped in cement tooth root, and others - in the periosteum of the alveolar process. What kind of fibers they are? A * Sharpey fibers B Korff fibers C Ebner fibers D Purkinje fibers E Argyrophilic fibers

On the histological slide were observed around the tooth periodontal thick bundles of collagen fibers that provide fixing the tooth in the dental alveoli. What are these fibers in the cervical area of the tooth? A * Circular B Oblique C Apical D Elastic E Myelinated

On the histological slide of periodont were observed dense connective tissue composed of thick bundles of collagen fibers and provides a fastening tooth in the dental alveoli. This structurec has a special name: A * Dental fiber connections B Breakthrough fiber C Myelinated fibers D Korff fibers E Ebner fibers

42 years old patient, addressed to the dentist complaining of severe toothache. After examining the

57 patient the doctor found inflammation of the dental pulp. What tissue forms the pulp of the tooth? A * Loose connective tissue B Amorphous dense fibrous connective tissue C Dense fibrous connective tissue framed D Reticular connective tissue E Mesenchyme

To keep the tooth in the hole in the periodontal alveolar processes are bundles of collagen fibers that are arranged in different directions. How periodontal fibers oriented on the sides of the root? A * Obliquely B Vertical C Horizontal D Perpendicular E Circularly

At the main part of the crown, neck and root of the tooth are present a dentin, whose length may increase with age, possibly as part of his recovery from injury. What structures provide these processes? A * Odontoblasts B Dentinal tubules C Perytubular dentin D E Cementoblasts

In the molars is visible a tissue, located at the upper part of the roots and in places of their branching. The tissue contains cells lying in lacunae and numerous collagen fibers with radial or longitudinal direction. Name this tissue. A * Cell cement B Reticular fibrous bone tissue C Dentin D Enamel E Dense connective tissue

In the enamel on the edge of the dentin found non calcified areas that are often a place of infection in the tooth. How to call such structures? A * Enamel bundles B Enamel prisms C Enameloblasts D Dentynoblasts E Toms Fibers

In histological slide, showing the histogenesis of tooth visible deposition of hydroxyapatite crystals in the form of globules. For what tooth characteristic this type of mineralization? A * Dentin B Enamels C Periodontal D Cement E Pulp

The child complains of a toothache. Dentist stated carious enamel damage. Number of minerals which decreases in caries damage: A * Phosphorous, fluorine and calcium; B Sodium, calcium, potassium, C Potassium, phosphorus, fluorine; D Magnesium fluoride, calcium

58 E Phosphorus, magnesium, potassium

On histological slide grinding tooth enamel defined light and dark bands width 100 mm, directed radially. Identify the formation of enamel. A *Bands of Gunther Shreher B Lines of Retius C Perykimatiy D Enamel prisms E Enamel tufts

In histological slides were found acellular cement. In which part of the tooth localized this tissue? A *On the lateral surface of the tooth root B On the surface of the crown C Forms a layer of coronal pulp D At the top of the tooth root E In the pulp canal

In the investigation of the chemical composition of dentin was found that in some areas it contains high content of minerals. Any damage to tooth decay much faster it breaks down, leading to the extension tubes and increasing the permeability of dentin. What type of dentin is it? A * Peritubular dentin B Interglodular dentin C Predentine D Mantle dentin E Circumpulpal dentin

On histological slide grinding crown dentine in the intercellular substance is determined by a small number of collagen fibers (fibers of Korf) coming in the radial direction. Name this layer of dentin. A * Mantle dentine B Circumpulpal dentin. C Granular layer. D Interglobular dentin. E Predentine.

In histological preparation grinding teeth is determined by cell-free tissue, consisting of intercellular substance pierced tubes, which are located in the cell processes. What structure is represented in the sample? A * Dentine. B Enamel. C Pulp. D Cement. E Dense connective tissue.

Electron photomicrographs of transverse thin sections of tooth enamel lesions appear oval, polygonal or arched form, consisting of compacted and organized hydroxyapatite crystals. What is this formation A * Enamel prisms B Lines of Retius C Bands of Gunther - Shreher D Perykimatium. E Collagen fibers.

On histological slide of the tooth in one of the tissues in the intercellular substance visible collagen fibers with radial and tangential direction. Decide for which tissue histogenesis is this a typical picture? A * Dentin

59 B Enamel C Cementum D Pulp. E Dense connective tissue.

In longitudinal grinding tooth dentin tubules seen. What is inside the tubules? A * Processes of dentynoblasts. B Processes of enameloblasts. C Body of dentynoblasts. D Fbroblasts. E Elastic fibers

Before teeth come out first on their roots appears a solid tissue that looks like membrane reticulated bone. What tissue is it? A *Cement B Dentin C Enamel D Loose fibrous connective tissue E Dense fibrous connective tissue

A histological specimen presenting a tooth slice shows that the intercellular dentin substance contains collagen fibers being tangential to the dentinoenamel junction and perpendicular to the dentinal tubules (Ebner's fibers). This dentin layer is called: A *Parapulpar dentin B Mantle dentin C Granular layer D Interglobular dentin E Secondary dentin

Histological examination of trasverse enamel slice revealed linear banding in form of concentric circles that is pointing at an angle to the dentinoenamel junction. Name these structures: A *Retsius' lines B Hunter-Schreger's lines C Enamel plates D Enamel fascicles E Enamel spindles

A histological specimen presents a developed tooth that has a coating resistant to acids, but it can be found only on the lateral surfaces of the tooth. What coating is meant? A *Cuticle B Dentine C Enamel pellicle D Enamel E Cement

Histological study of an extirpated pulp revealed some cylindrical cells in its peripheral layer. What are these cells called? A *Odontoblasts B Fibroblasts C Monocytes D Ameloblasts E Myofibroblasts

Secretory activity of odontoblasts was damaged in a milk tooth during the formation of mantile dentin.

60 Determine which fiber formation will change? A.*Korff fibers B. Reticular fibers C. Elastic fibers D. Ebner fibers E. Nervous fibers

Irregular dentinal tubules and collagen fibrils are found in a decalcified adult tooth. Name this kind of dentin? A. *Secondary dentine B. Primary dentine C. irregular dentine D. Sclerotic dentine E. "Dead" tracts

Histological examination revealed enamel cross section linear striations in the form of concentric circles that are directed at an angle to the dentine enamel connection. What are these structures? A. *Lines of Retzius B. Bundles of Gunter-Shreher C. Enamel lamellae D. Enamel spindle E. Enamel bundles

Sensitive nerve endings in the form of glomeruli are found in the periodontal bundles along their fibers. What role do these receptors play? A. *Touch receptors B. Thermoreceptors C. Pain receptors D. Chemical receptors E. Mechanical receptors

Dark areas of dentine are detected in the section during a tooth extraction. A result of which process formed these areas? A. *Disorder of dentine B. Proliferationof dentinoblasts C. Hypertrophia of dentinoblasts D. Atrophy of dentinoblasts E. Proliferation of fibroblasts

Radial light bounds are found in thin sections of an elderly man's dentin. These areas are called? A.*Predentine B. "Dead tracts" C. Secondary dentine D. All E. Irregularly dentine

The excess filling in a cavity led to an overestimation of occlusion. This caused pain during bite due to periodontal injury. What kind of nerve endings in the periodontum are involved in feeling pain? A. *Free nerve endings B. Absence of capsule C. Presence of capsule D. Axo-muscle E. Synapses

61

Hertwig's vagina around the tooth is damaged 5 months after birth. Development of which tooth layer prevents this fact? A. *Cementum B. Enamel papillae C. Enamel saccule D. Pulp E. Dentine

For some reason, the effectiveness of particular cells in the marginal zone of the pulp has been temporarily inhibited. What tooth structure is under risk of shortage of its physiological regeneration? A.*Dentine B. Enamel C. Pulp D. Cellular cement formation E. Acellular cement formation

From two tooth specimens, was histologically detected that: in one -acellular cement, and in the second - cellular. From which section of the tooth was the second specimen prepared from? A. *Root apex B. Cervix of tooth C. The upper region of the tooth below the gumline D. Crown of tooth E. The bound between crown and root

A student made a mistake while answering to a question about the composition of periodontum due to calling out Merkel's cartilage tissue and tooth pouch. What should be the correct answer? A. *Periodontum, gingival, alveolar processus B. Pulp, apex, periodontum C. Periodontum, cementum, alveolar processus D. Gingiva, papillae of gingiva, alveolar processus E.

In the course of life, the crown is continually filed down. What mechanism maintains the sustainability of the overall length of the tooth throughout life? A. *Growth of roots B. Formation of dentine C. Growth of periodontum D. Reducing the pulp chamber E. Growth of enamel

During the study of the tooth structure of a 40-year old individual, the dentin-enamel margin defined optically dense linear structure up to 1/3 the thickness of the enamel fusiform. What kind of structure did the researcher discover? A. *Enamel spindles B. Denticles C. Growth of periodontum D. Reducing the pulp chamber E. Growth of enamel

A secondary dentine is developed after an inflammation at the root of the tooth. It is characterized by the following features: A. *Interglobular dentine B. Similar to dentine

62 C. Don't differ from primary D. Appear after tooth emption E. Appear during histogenesis process

Prismatic cells that have hexagonal sections and nucleus located in the central part of the cell appear during the development of tooth . What are these cells? A. *Preenameloblasts B. Exterior enameloblast nucleus C. Cambial cells D. Enamel pulp cells E. Preodontoblasts

The pulp of the tooth is an area in which blood vessels and nerves are surrounded by collagen and reticular fibers, fibroblasts, macrophages, undifferentiated cells -pericytes, adventitial cells, tissue basophils. What is the area of the pulp? A. *Central layer of pulp B. Peripheral layer of pulp C. Predentine layer of pulp D. Coronal pulp E. Pulp stones

During an examination, the dentist found an oral disease -tooth decay. What dental caries structure is affected first? A. *Enamel B. Cuticle C. Pellicle D. Pulp E. Dentine

In a histological section of an embryo's mandible reveals tooth germ at the stage of histogenesis of tooth tissues. What tissue is formed first? A. *Dentine B. Enamel C. Cementum D. Pulp E. Periodontum

On a specimen from the cross-section of tooth was microscopically detected pulp, dentin and stria, which is similar to the rough fibers of bone, but without cellular elements. At what level was the cut made? A. *At the level of root B. At the level of root apex C. At the level of cervix D. At the level of crown E. At the level of crown apex

On a microspecimen of the root of tooth revealed the absence of cells of the outer layer of the cement's surface. From what tissues does this pathological changes relate to? A. *Cellular cementum B. Acellukar cementum C. Dentine D. Enamel E. Wall of alveolus

63 On a microspecimen of the peripheral zone of the tooth's pulp was revealed abnormal pear-shaped cells with a single branched long process, located here several layers. what tissue of the tooth according to this was damaged? A. *Dentine B. Enamel C. CEmentum D. Bone of alveolus E. Periodontum

On a microspecimen of the longitudinal section of the tooth, we distinguish hard and loose tissues. Which of these tissues how ectodermal origin? A. *Enamel B. Dentine C. Cementum D. Pulp E. Periodontum

During microscopic examination of the tooth's crown, enamel pellicle is diagnosed. Which structural components are part of the enamel's pellicle? A. *Thin layer of glycoprotein B. Rete of collagen fibers C. Accumulation of calcium salts D. Residual of enaml organ cells E. Gingiva

A study was conducted to examine the source of tissues which supply the tooth. What structural component of the tooth provides dentin trophic? A. *Pulp B. Enamel C. Cementum D. Periodontum E. Bone of processus maxillae

In the thin sections of the crown we can see structures that are called -enamel tufts. How are they formed? A. *Processus of cuticle enamel B. Enamel prisms C. Fibers with inorganic substances D. Fibers with organic substances E.

On a histological preparation are tooth structures that form cement. Which cell are part of the formation of tooth cement? A. *Cementoblasts B. Odontoblasts C. Enameloblasts D. Cementocytes E. Osteoblasts

Numerous processes of dentinoblasts with the help of intercellular contacts are interconnected. Processes are capable of reduction, thereby providing tissue fluid circulation and saturation of mineral substances of dentine and enamel. Due to which organelles forming part of the dentinoblast processes cause thus reduction? A.*Microfilaments

64 B. Golgi apparatus C. Mitochondria D. Lysosomes E. Ribosomes

A study was conducted t determine the tissue structures belonging to the tooth. What tissue belongs to the periodontum? A. *Dense regular connective tissue B. Loose connective tissue C. Bone tissue D. Reticular tissue E. Adipose tissue

In the thin sections of tooth enamel were diagnosed Retzius lines. What factor determines the emergence of parallel lines of enamel (Retzius lines)? A. *Periodicity of enlargement and calcification prisms B. Dilation of enamelprisms C. Elongation of enamel prisms D. Perioicity appearance of interprismatic material E. Periodicity appearance enamel lamellae

A 43 year old patient for a long time has not had enough vitamin C in his diet. What is the pathology of the supporting apparatus of the tooth to be expected in the first place? A. *Periodontum disorder B. Keratinization of gingival epithelium C. Gingival pockets formatia D. Alveolar bone transformation E. Disorder of sulcus epithelium

Tooth development.

During examination of a child’s oral cavity a pediatrician found 8 incisors. The child’s development corresponds to his age. How old is the child? A. *10-12 month B. 6-7 month C. 7-8 month D. 12-15 month E. 16-20 month

Study of the histological specimen of a baby’s primary toothrevealed hypoplasia (underdevelopment) of enamel/ this abnormality is caused by the disruptions in the activity of the following cells: A. * B. Pulp cells of the enamel organ C. Outer enamel epitelium D. Cells of the of the enamel organ E. Odontoblasts

In histological slide of tooth germ enamel organ outer surface is rough, the cells of the inner layer of cells occurred polarity (inversion nuclei). Determine the beginning of the process which preceded these changes. A * Enamelogenesis B Dentinogenesis C Pulpogenesis D

65 E Periodontal development

AQ 36-year-old-patient with a several year history of chronic pulpitis had undergone tooth extraction. Microscopic examination of the pulp revealed some deep purple structurulles areas which can be interpreted as: A. *Dystrophic calcification B. Metabolic calcification C. Metastatic calcification D. Lithiasis E. Dental tartar

During the examination of the patient was found abnormalities of enamel. With damage of what structural components of tooth germ is it occurs? A * Inner epithelium of enamel organ B Intermediate layer of the enamel organ C Pulp of enamel organ D Outer epithelium of enamel organ E Neck of enamel organ

During the development of the primary tooth dentin tissue is laid. What is the source of its development? A * Dental papilla B Dental pouch C Tooth plate D Inner cells of the enamel organ E Outer cells of the enamel organ

In the study of histological slides dental pulp was observed in the connective tissue dominated by bundles of collagen fibers odontoblasts layer thin intermediate layer is weak. In which area of the tooth pulp has the following features? A * Root pulp B Coronal pulp C Layer Weil D Peripheral layer of pulp E The central layer of pulp

On histological slide of the tooth is determined enamel organ in the form of "bell", which shows the external cube-shaped enamel cells, high internal prismatic cells and centrally located cells with processes that form a network. What is the period of tooth represented in the sample? A * During the formation and differentiation of dental germs. B Period bookmarks dental germs. C During the formation of the tissues of the tooth crown. D During the formation of the tissues of the tooth root. E Odontiasis period.

During the tooth development the enamel organ has prismatic cells with hexagonal intersection; the nucleus is situated in the central part of the cell. What cells are meant? A *Preenameloblasts B Exterior enameloblasts C Cambial cells D Enamel pulp cells E Preodontoblasts

During the embryogenesis of oral cavity the development of dental enamel was disturbed. What source of dental development was damaged?

66 A *Epithelium B Mesenchyma C Mesoderma D Dental saccule E Dental papilla

A histological specimen of mandible of an embryo shows a tooth germ with the dental papilla made up of small stellate basophilic cells. What tissue forms this part of the tooth germ? A *Mesenchyme B Epithelial C Reticular D Cartilaginous E Osseous

In the process of tooth tissue histogenesis dentin wasn't formed in time for some reasons. What process of further histogenesis will be delayed or will not take place at all? A *Enamel formation B Pulp formation C Predentinal space formation D Cellular cement formation E Acellular cement formation

In animal experiments the dental germs destroyed the inner layer of the epithelium of the enamel organ. Development of which tooth is broken? A * Enamel B Dentine C Cementum D Pulp E Periodontal ligament

During embryonic development the dental epithelial's dental papilla mesenchymal cells surface was damaged. This can lead to the damage of formation of which tooth structures? A. *Dentine B. Enamel C. Cementum D. Periodontum E. Enamel cuticle

Internal dental sac cells were damaged during tooth morphogenesis. The formation of which tooth structures will be damaged? A. *Cementum B. Enamel C. Dentine D. Pulp E. Periodontum

After a radiograph an 11 year old child's lateral incisors are absent. This is connected with? A.*Disorder the formation of enamel organs B. Disorder the formation of enamel saccule C. Disorder the formation of enamel papilla D. Disorder the formation of dentine E. Disorder the formation of cementum

67 In embryogenesis the oral tooth enamel develops from: A. *Epithelial B. Mesenchyme C. Ectoderma D. Enamel saccule E. Enamel papilla

The most common places of infection in the tooth enamel are areas with few inorganic components that penetrate the entire thickness of the enamel, dividing it into segments. Name these entities: A. *Enamel lamellae B. Enamel bundles C. Enamel prisms D. Enameloblasts E. Tomes process

Numerous receptors were found along the periodontal bundles of fibrous clumps of fibers. What functional role do they play? A. *Pressure receptors B. Chemical receptors C. Pain receptors D. Thermoreceptors E. Proprioreceptors Which cells during tooth development are characterized by inversion (organelles and nucleus move to opposite side), and change the polarity? A. *Enameloblasts B. Odontoblasts C. Preodontoblasts D. Cementoblasts E. Cementocytes

In embrogenesis the oral periodontum develops from: A. *Enamel saccule B. Enamel papillae C. Enamel lamilla D. Paraenamelobasts E. Enameloblasts

Harmful irritation of the tissue of the tooth resulted in the formation of denticular structures across the peripheral zone of the pulp. How does this phenomenon influence the existence of the tooth? A. *Loss of dentine regeneration ability B. Loss of pulp regeneration ability C. Loss of cementum regeneration ability D. Loss of tooth innervation E. Loss of enamel regeneration ability

In the sixth month after the birth of a baby, begun milk , all the tissues of the tooth were formed in the least from? A. *Cementum B. Enamel C. Dentine D. Peripheral layer of pulp E. Central layer of pulp

A later of odontoblasts, placed in the peripheral layer of pulp is able to perform the barrier function

68 between predentin and pulp. Due to presence of what structures is this possible? A.*Presence of the light junction B. Presence of desmosomes C. Presence of synapses D. Presence of Tormis processus E. Presence of RER

The final maturation of the enamel after is entitled(called): A. *Tertiary mineralization B. Secondary mineralization C. Primary mineralization D. Desquamation E. Secretion

During enamel formation, the process of its extraction from water and protein was being damaged. At what stage of enamel development does this happen? A. *Maturation (secondary mineralization) B. Secretion (Primary mineralization) C. Formation of secretory D. Formation of maturation ameloblast E. Formation of dental germs

On an electron microphotography is presented interprism enamel. Which cells from it? A.*Secretory ameloblast B. Enameloblasts II C. Enameloblasts I D. Preenameloblasts E. Secretory active

During the process of histogenesis, for some reason, the dentine was not formed in time. What further process of histogenesis did not take place or was removed in time? A. *Formation of enamel B. Formation of pulp C. Formation of interglobular dentine D. Formation of cellular cementum E. Formation of acellular cementum

A longitudinal section of an extracted tooth was made. On this section was observed pathological changes in the soft tissues of dental pulp, which is inherent in such cells except: A. *Osteocytes B. Fibroblat C. Macrophages D. Adventitial E. Dentinoblasts

Hypoplasia (underdevelopment) of enamel was found in a milk tooth histological specimen. Determine, the activities of which cells this damage related to: A. *Inner enamel B. Outer enamel C. Cells of dental papilla organ D. Cells of enamel knot E. Odontoblats

During the study of a histologcal specimen, a baby's tooth revealed a strict dark line that seperates the

69 enamel that formed after birth. Name this structure? A.*Neonatal line B. Enamel lamellae C. Lines of Retzius D. Enamel spindles E. Enamel fascicles

On an electronic microphotography of enamel organ reveals a prismatic cell with developed granular endoplasmic reticulum and Golgi complex. At the apical part of the cell -Tom's process containing secretory granules and small vesicles. Identify this cell? A. *Ameloblast secretory active B. Preenameloblast C. Outer layer cells of enamel organ D. Cells of pulp enamel E. Cells of enamel organ

In the cement carrries were observed, destructive changes in the cement; its resorption. Specify the origin of the cement's development? A. *Inner cells of dentes sacculus B. Outer cells of dentes sacculus C. Intermediate cells of enamel organ D. Inner cells of enamel organ E. Outer cells of enamel organ

An electron microphotography defines dental papilla cells with an oval nucleus rich in Euchromatin. In the cell's cytoplasm are developed granular endoplasmic reticulum, Golgi apparatus. The apex contains process with lateral branches. Identify this cell? A. *Odontoblast B. Cementoblast C. Mesenchyme cells D. Enameloblasts E. Fibrobast

Histological specimen of the mandible reveals 10 tooth buds associated with dental plate. Which element of the tooth's germ will develop from them? A. *Enamel organ B. Enamel papillae C. Enamel saccule D. Enamel fascicles E. Enamel pearls during histological examination, the peripheral layer of a pulp defines cells of cylindrical forms. What is the name of these cells? A. *Odontoblasts B. Fibroblasts C. Monocytes D. Ameloblasts E. Myofibroblasts

A study was conducted to study the peculiarities of tooth structures. Which component of the tooth contain blood vessels? A. *Pulp B. Enamel cuticle C. Acellular cement formation

70 D. Dentinal tubules E. Cellular cement formation

On a histological specimen of an embryo's mandible reveals a dental epithelial organ. What are the components of this germ? A. *External, internal and intermediate cells B. External cells C. Intermediate cells D. Internal cells E. Peripheral cells

Esophagus. Stomach.

The man of 35 years with a stomach ulcer the resection of antral department of stomach is made. Which secretion of gastrointestinal hormone as a result of operation will be broken most of all? A. Gastrin* B. Secretin C. Neurotenzin D. Hystamine E. Holecystokinin

At the patient at gastroscopy the insufficient quantity amount of mucus on a surface of mucosa is revealed. To what infringement of function of stomach’s cells it is connected? A. Parietal cells B. Columnar cells* C. Endocrinocytes D. Cervical cells E. Chief cells

On histologic preparation the cross-section of alimentary tubular organ’s wall which mucous shall is covered by nonkeratinazed epithelia is submitted. What is the organ? A. Uterus B. Duodenun C. Small intestine D. Esophagus * E. Appendix

On the fourth week of embryonic developments occurs physiological atresia of esophagus. Up to the end of the eighth week it again becomes passable. What biological process provides recanalisation of esophagus at human embryo? A. Descvamation B. Меiosis C. Necrosis D. Apoptosis* E. Mitosis

During histological examination of the stomach it was found out a significant reduction or complete absence of parietal cells in the glands. Mucose membrane of what part of the stomach was studied? A Pyloric part* B Fundus of stomach C Cardia D Body of stomach E -

71 When the pH level of the stomach lumen decreases to less than 3, the antrum of the stomach releases peptide that acts in paracrine fashion to inhibit gastrin release. This peptide is: A GIF * B Acetylcholine C Gastrin-releasing peptide (GRP) D Somatostatin E Vasoactive intestinal peptide (VIP)

Examination of a 43 y.o. patient revealed that his stomach has difficulties with digestion of protein food. Gastric juice analysis revealed low acidity. Function of which gastric cells is disturbed in this case? A Parietal exocrinocytes * B Main exocrinocytes C Mucous cells (mucocytes) D Endocrinous cells E Cervical mucocytes

An electron microphotography of a fragment of proper gastric gland shows a big irregular round-shaped cell. There are a lot of intracellular tubules and mitochondria in the cytoplasm. Specify this cell: A Parietal cell * B Principal cell C Undifferentiated cell D Mucous cell E Endocrine cell

A patient ill with chronic gastritis went for endogastric pH-metry that allowed revealing decreased acidity of gastric juice. It is indicative of diminished function of the following cells: A Parietal exocrinocytes * B Chief exocrinocytes C Endocrinocytes D Cervical cells E Accessory cells

A patient underwent gastroscopy that revealed insufficient amount of mucus covering the mucous membrane. This phenomenon is caused by the dysfunction of the following cells of stomach wall: A *Cells of prysmatic glandular epithelium B Parietal cells of gastric glands C Principal exocrinocytes of gastric glands D Cervical cells of gastric glands E Endocrinocytes

The 60 years old patient is suffering from chronic gastritis. At the endoscopy of the stomach observed changes in the epithelium of the mucosa. Which type of epithelium covers of the stomach mucosa? A * Simple columnar mucous epithelium B Pseudostratified ciliated columnar epithelium C Simple squamous epithelium D Simple cuboidal epithelium E Simple columnar brush-border epithelium

The patients with burnings of the esophagus was examined by the doctor and found that lesions of the mucous membrane are not deep. Due to which layer of cells will take place regeneration of damaged epithelium. A * Basal B Spinous C Grained

72 D Intermediate E Surface

On histological slide represented a cross section of a hollow organ wall, which has a mucous membrane consist of nonkeratinized stratified epithelium. What kind of structure is it? A * Esophagus B Duodenum C Colon D Uterus E Appendix

On microscopic examination were presented striated muscle tissue of the digestive system. In what kind of organ the biopsy was taken? A * Esophagus B Stomach C Duodenum D Ileum E Appendix

A malignant tumor which developed from the transverse striated muscle of a digestive system organ was submitted for pathologic anatomic research. The biopsy was taken from which organ? A. *Upper portion of esophagus B. Stomach C. Duodenum D. Ileum E. Lower portion of esophagus

During microscopic research of the esophagus it was discovered that it has a mucous membrane that is covered with a stratified squamous non-keratinized epithelium and a lamina propria that contains simple tubular glands, definitive sections of which consist of mucous and a few parietal cells. What organs is it? A. *Esophagus B. Stomach C. Small intestine D. Trachea E. Urethra

A digestive tract organ was selected for histological research. The mucosal and submucosal membranes form longitudinal folds. The superficial surface of the mucous membrane is smooth and lined by a Stratified squamous non-keratinized epithelium. Define this organ? A. *Esophagus B. Stomach C. Duodenum D. Large intestine (colon) E. Trachea

In a histological investigation of the neck region of the stomach's proper gland we identify small cells with a high nuclear-cytoplasmic ratio and mitotic figures. Define the function of these cells? A. *Regeneration of glandular epithelium B. Protective C. Endocrine D. Ion Cl- decretion E. Pepsinogen secretion

Under the action of harmful factors occurred focal damage of the epithelium of the stomach. What kind

73 of cell can provide regeneration process? A * Mucous neck cells B Parietal cells C Chief cells D Enteroendocrine cells E Mucocytes

On the electronic microphotographs of fundic glands were determined a large, pale and round to pyramidal cells. They have one or 2 central nuclei and acidophilic cytoplasm. The many mitochondria indicate that their secretory activity is energy-dependent. What kind of cell they are? A * Parietal (oxyntic) cells B Mucous neck cells C Undifferentiated cells D Enteroendocrine cells E Chief cells

Insulin injection did for assess the completeness of vagotomy accompanied by a significant increase of ph of gastric juice. Which cells of gastric glands controls this process? A * Parietal cells B Enteroendocrine cells C Chief cells D Mucous neck cells E Undifferentiated cells

On the histological sections of fundic glands we can see as large cells with acidophilic cytoplasm. What kind of component of gastric juice is produced as a result of the activity of these cells? A * Hydrochloric acid B Pepsinogen C Mucus D Serotonin E Gastrin

The 20 years old patient suffers from rheumatism appointed of prolonged use of aspirin. What is the structural component of the mucous membrane of the stomach is best suited to provide her protection from damage? A * Simple columnar mucous epithelium B Connective tissue C Muscle tissue D Multi-ciliated epithelium E Stratified squamous nonkeratinized epithelium

The patient biopsy from the stomach shows histologically revealed a significant reduction or complete absence of parietal cells in the glands. Which layer of mucous membrane they studied? A * Pylorus B Fundus of stomach C Cardiac part D Body of stomach E –

In 42 years old patient after radiotherapy of cancer of the stomach developed pernicious anemia due to damage cells by producing intrinsic factor. Which cells of fundic glands were damaged? A * Parietal cells B Mucous neck cells C Surface mucous cells

74 D Enteroendocrine cells E Chief cells

In the embryonic material was damaged endoderm. What kind of changes of development may arise in this process? A * Stomach B Heart C Kidneys D Aorta E Salivary glands

Damaged by exposure to various factors of gastric mucosa can restore its integrity. By which cells of gastric glands can occur their regeneration? A * Mucous neck cells B Parietal cells C Paneth cells D Chief cells E Enteroendocrine cells

By ingestion of a foreign body in the stomach was damaged epithelium of the child. How the cell regeneration process possible? A * Mucous neck cells B Chief cells C Parietal cells D Connective tissue cells E Fat cells

During the fibrogastroscopy of the patient revealed damage of the mucosa layer. By which cells will occur in the treatment of epithelial regeneration? A * Poorly differentiated mucous neck cells B Chief cells C Parietal cells D Enteroendocrine cells E Paneth cells

During inflammatory diseases of the stomach damaged surface epithelium of the gastric mucosa. What kind of epithelium was damaged? A * Simple columnar mucous epithelium B Stratified squamous nonkeratinized epithelium C Stratified squamous keratinized epithelium D Simple cuboidal epithelium E Stratified cuboidal epithelium

Analysis of biopsy material of human gastric mucosa, gastritis patient showed a dramatic decrease in the number of parietal cells. How will change the components of gastric juice? A * Reduce acidity B Increased acidity C Increased gastric juice D Reduction of gastric juice E Reducing mucus production

Examination of a patient, suffering from atrophic gastritis, revealed megaloblastic anemia. The anemia is likely to be caused by the deficiency of the following substance: A *Gastromucoproteid

75 B Vitamin B6 C Vitamin B1 D Iron E Erythropoietins

A patient underwent gastroscopy that revealed insufficient amount of mucus covering the mucous membrane. This phenomenon is caused by the dysfunction of the following cells of stomach wall: A *Cells of prysmatic glandular epithelium B Parietal cells of gastric glands C Principal exocrinocytes of gastric glands D Cervical cells of gastric glands E Endocrinocytes

During the examination of the patient's oral cavity dentist noticed that his tongue is rough hypertrophic nipples, deep furrows. The doctor advised the patient to consult a gastroenterologist. The examination revealed that he had considerably increased acidity of gastric juice. Hyperfunction of which cells in the glands of the mucous membrane of the stomach mainly caused this condition? A * Parietal cells of fundic glands B Chief cells C Additional mucocytes D Goblet cells E Exocrine pancreatocytes

The 60 years old patient is suffering from chronic gastritis. During an endoscopy of the stomach observed changes in the epithelium of the mucosa. What epithelium has undergone a change? A * Simple columnar mucous epithelium B Connective tissue C Muscle tissue D Multi-ciliated epithelium E Stratified squamous nonkeratinized epithelium

A patient with hypersecretion of a gastric juice is advised to stay away from a diet rich in broth and vegetable broth, because they stimulate gastric secretion due to the following mechanism: A. *Stimulate gastrin production by endocrine cells B. Irritate taste buds C. Irritate mechanic receptors of oral cavity D. Irritate mechanic receptors of stomach E. Stimulate stimulating production in duodenum

A resection of the stomach with the removal of pyloric substance was performed. What process of the GIT will be affected? A. *Chime transit into duodenum B. Intestine peristaltic C. Reabsorption D. Juice secetion in duodenum E. -

A 45 year old man has complaints about the dysfunction of his stomach. During a complex investigation a tumor was found in the epithelial tissue. What cells gave development to this tumor? A. *Mucous neck cells B. Chief (zymogenic) C. Enteroendocrine cells D. Surface mucous cells E. Parietal cells

76

The glands at the bottom of the stomach contain cells, which with their own secretions weaken or strengthen the formation of gastric juice components, gastric motility and activity of the pancreas. What are these cells? A. *Gastropancreatic endocrine B. Chief zymogenic cells C. Parietal cells D. Simple columnar cells E. Mucous neck cells

Due to the deficit synthesis of intrinsic factor (gastromucoproteid) we detect pernicious anaemia of Adison-Beimer in a patient, which cells of the principal glands in a stomach are damaged A. *Parietal B. Chief(zymogenic) cells C. Mucous neck cells D. Endocrine cells E. Surface mucous cells

During microscopic investigation we observe a digestive tract organ, relief of which presents folds and fields. What epithelium is covering the mucous membrane of this organ? A. *Simple columnar epithelium B. Simple cuboidal C. Simple squamous D. Striated non keratinized squamous E. Pseudostratified columnar epithelium

An organ is selected for morphological research. The wall of which consists of a mucosal, submucosal, muscular and serous layers. Name this organ? A. *Stomach B. Esophagus C. Small intestine D. Large intestine(colon) E. Urethra

After an intragastric Ph measurement a patient with chronic gastritis is diagnosed with the reduced acidity of gastric juice. What cell function is reduced? A. *Parietal B. Chief C. Enteroendocrine D. Mucous neck cells E. -

Pepsinogen is produced by the stomach's principal cells. However secretions of other cells of the stomach are required in order to activate pepsinogen in the cavity of the stomach. What is the name of these cells? A. *Parietal B. Chief C. Mucous neck cells D. Enterendocrine cells E. Mucous cells

Small and large intestine. What deficiency of enzyme is the reason of incomplete digestion of fats in a gastroenteric path and increases in quantity amount of neutral fat in feces more often?

77 A. Lipase* B. Enterokinase C. Lactase D. Dipepttidase E. Secretine

On histologic section of small intestine’s wall at the crypts’ cells located by groups are found, in apical parts contain big acidophilic secretory granules; cytoplasm is basophilic. What are these cells? A. Columnar cells B. Paneth cells * C. Endocrinocytes D. Goblet cells E. Enterocytes

At the patient with chronic enterocolitis (the inflammation of intestines) is revealed infringement of digestion and absorption digestive products in thin gut as a result of insufficient quantity amount in intestinal juice dipeptidases. In what cells synthesis of these enzymes is broken? A. Goblet cells B. Columar cells without brush border C. Paneth cells * D. Stem cells E. Enterocytes with a brush border

The patient with thyreotoxicosis complains of diarrhea, heavy feeling in stomach. At examination - feces without pathological changes. At radiological research definitely acceleration of passage of baric masses along intestines. The hypertonus of what shall of alimentary tube is the reason of the given condition? A. Serous B. Mucous C. Adventitial D. Submucous E. Muscular*

An electron microphotograph of duodenal epithelium clearly shows a cell with electron-dense granules in the basal pole. What cell is it? A. *Endocrine B. Prismatic with a limbus C. Poorly differentiated D. Goblet E. Parietal

The intestinal submucosal membrane is being investigated, complex branched tubular glands resembling the pyloric stomach glands are found. Which part of the intestine is found in the histological specimen? A. *Duodenum B. Ileum C. Rectum D. Appendix E. Sigmoid colon

In some diseases of the colon change correlation of number between epithelial cells of the mucosa. What types of cells predominate in the epithelium of the crypts of the colon normally? A * Goblet cells B Columnar villous epithelial cells C Endocrinocytes D Cells with acidophilic granules

78 E Undifferentiated cells

Some diseases of the small intestine associated with dysfunction of exocrynocytes with acidophilic granules (Paneth cells). Where are these cells located? A * At the bottom of intestinal crypts B On the apical side of intestinal villi C On the sides of the intestinal villi D In place of transition in villus crypt E In the upper part of the intestinal crypts

During endoscopic examination of the patients with chronic enterocolitis (inflammation of the colon), there is no specific structures reliefs of the small intestine. What components determine the relief features of the mucous membrane of this organ? A * Circular folds, villi and crypts B Felds, folds, holes C Haustry, villi, crypt D Oblique folds E Villi

During the examination of the patients with diseases of the small intestine revealed disruption of the wall and membrane digestion. With dysfunction of what kind of cells it is connecting? A * Column with border B Column without border C Goblet D Paneth cells E Endocrinocytes

During the diseases of the mucosa layer of the small intestine suffers absorption function. What kind of epithelium is responsible for this function? A * Simple columnar epithelium B Simple cuboidal C Simple columnar D Stratified squamous epithelium E Stratified cuboidal

The student had a histological slide of the small intestine submucosa in which the foundation has a large number of glands. Which part of the small intestine is it? A * Duodenum B Jejunum C Ileum D Ascending colon E Descending colon

On the electronic microphotogram of crypts of the small intestine can be identified some cells. They lying in the bases of the crypts, have many large acidofilic secretory granules. What is the name of these cells? A * Paneth cells B Enterocytes C Goblet cells D Undifferentiated cells E Enteroendocrine cells

On a histological specimen of the small intestine in the lamina propria of mucosa revealed clumps of cells spherical shape with large basophilic nuclei surrounded by a narrow rim of cytoplasm. In most of

79 the central part of the light clusters and contains fewer cells than peripheral. Which is the morphological structure of such clusters? A * Lymphatic nodule B Nerve bundle C Fat cells D Blood vessels E Lymphatic vessels

On a histological specimen of submucosa of the small intestine is filled of endings of the protein secretory glands. Where was the section of intestine done which is presented in the sample? A * Duodenum B Jejunum C Cecum D Ileum E Appendix

In the cytoplasm of epithelial cells of the colon’s crypts were found dypeptydase and lysozyme. What kind of cells can produce these enzymes? A * Paneth cells B Columnar epithelial cells C Goblet cells D A-cells E S-cells

On a histological specimen of a wall of the digestive system in the lamina propria of the mucosa and submucosa were found numerous lymphoid nodules. Name the structure. A *Appendix B Stomach C Duodenum D Ileum E Colon

During a biopsy investigation of the wall of the small intestine has been taken out the part of mucosa layer. What epithelium covers the mucosal surface of the organ? A * Simple columnar epithelium with goblet cells B Stratified squamous nonkeratinized epithelium C Stratified squamous keratinized epithelium D Simple cuboidal epithelium E Stratified cuboidal epithelium

A specimen presented an organ of the digestive system which has a thickened lamina propria of the mucous membrane, numerous lymphoid nodules and a submucosal membrane. In what organ do the lymphoid nodules occupy the largest volume in relation to the thickness of the cell? A. *Appendix B. Stomach C. Duodenum D. Ileum E. Colon

A patient with polypous columnar growths in the rectal area was examined by a proctologist. Which epithelium became the source of polyps? A. *Stratified squamous non keratinzed B. Simple cubic C. Simple flat

80 D. Simple cubic E. Simple polynuclear

During an endoscopic examination of active digestion we observe active movement of villi of the small intestine, as a result of which their length changes. Which of the following is the reason for this change? A. *Contraction of smooth muscle cells B. Plicae C. Peristaltic D. Pre-innervation E. Crypt enlongation

It is recommended to take drugs that are in the form of alcohol tincture before a meal. This applies especially to meals rich with fats. This is related to which functional features of the stomach? A. *Absorption function B. Splitting of nutrients C. Excretory function D. Endocrine E. All of the above

While describing a specimen of a particular organ of the gastrointestinal tract, a student noticed that within the lamina propria of the mucosal and submucosal membranes there were clusters of lymph nodes which had almost no crypts and few microvilli. Which organ has these features? A. *Ileum B. Jejenum C. Colon D. Appendix E. Stomach

The cells making up the mucous membrane of the intestine have borders on their surfaces(microvilli). In some diseases these borders are destroyed. Which function of the cell will be disturbed the most? A. *Absorption B. Digestive C. Excretion D. Accumulation E. Synthetic

An organ of the digestive tract is revealed in a histological specimen, villi and crypts are present in it's topography. What epithelium is covering the mucous membrane of the organ? A. *Simple prismatic fringularis B. Simple prismatic glandular C. Simple polynucleus prismatic ciliated D. Stratified squamous nonkeratinized E. Stratified transitive

The duodenal contents of a 36 year old woman is being researched. Which type of epithelium covering the gall bladder can be found in the sediment of the examinations? A. *Simple prismatic with border B. Simple cubic C. Simple prismatic ciliated D. Strtified flat E. Stratified cubic

In the diseases of small intestine the mucous membranes absorption function is damaged. What epithelium is responsible for this function?

81 A. *Simple prismatic with border B. Simple cubic C. Simple prismatic ciliated D. Stratified flat E. Stratified cubic

Salivary glands.

At micropreparation of the submandibular salivary gland there are shown some basket-shaped cells concentrated around the acines and excretory ducts. These cells surround bases of the serous cells and are called myoepitheliocytes. These cells relate to the following tissue: A Muscular tissue * B Epithelial tissue C Neural tissue D Special connective tissue E Loose fibrous connective tissue

What substance makes the saliva viscous and mucous and performs protective function, including protection from mechanical injury of mouth mucous membrane? A *Mucin B Glucose C Kallikrein D Amylase E Lysozyme

Acini containing 10-15 conical cells with a basophilic cytoplasm, round nucleus and a well developed granular endoplasmic reticulum are found in a specimen taken from an unkwown body. Acinus is surrounded by a basal membrane which when split contains myoepithelial cells. Cut is from which organ? A. *Parotid salivary glands B. Pancreas C. Lungs D. Sublingual E. Liver

As a result of development anomaly a newborn has malformation of major salivary glands. This anomaly is caused by damage of the following embryonic structure: A. *Ectoderm B. Splanchnotom C. Somites D. Entoderm E. Mesenchyme

A histological specimen of a mandibular gland shows an exctretory duct. Mucous membrane of the duct is lined with cubic epithelium whose cells have weakly developed organellas. What excretory duct is it? A *Intercalated B Striated C Interlobular D Common excretory E -

A microspecimen of parotid gland presents secretory acines with serous cells that synthesize mostly enzymes. According to the chemical composition classification, the parotid gland relates to the

82 following glands: A *Serous B Mucous C Seromucous D Enzymatic E -

Examination of a microspecimen made of an unknown organ revealed some acini that contained 10-15 cone cells with basophilic cytoplasm, round nucleus and well developed granular endoplasmic reticulum. An acinus is surrounded by a basal membrane with myoepithelial cells localized in its splitting. What organ is the slice made of? A *Parotid gland B Pancreas C Lungs D Sublingual gland E Liver

A microspecimen of the submandibular salivary gland shows some basket-shaped cells concentrated around the acines and excretory ducts. These cells surround bases of the serous cells and are called myoepitheliocytes. These cells relate to the following tissue: A *Muscular tissue B Epithelial tissue C Neural tissue D Special connective tissue E Loose fibrous connective tissue

It is known that the submandibular salivary gland has a slimy end sections, consisting of mucocytes. What are the signs characteristic of these cells? A * Flattened nucleus and light cytoplasm B Basophilic cytoplasm C Rounded nucleus in the center of the cell D Microvilli E Basal striation

Some diseases of the salivary glands caused by disruption of their excretory ducts. Which types are distinguished in the excretory ducts of salivary glands? A * Intra-, interlobular duct and common duct B Intralobular ducts, striated and common duct C Glistering, striated and common duct D Intra-, interlobular ducts E Intralobular ducts and outside the gland ducts

In histological slide of the gland determined only serous end sections. In the interlobular connective tissue visible ducts, lined with double-layer or multilayered epithelium. Determine this structure. A * Parotid gland B Submandibular salivary gland C Pancreas D Sublingual salivary gland E Liver

It is known that the secretion of parotid gland stands out in the mouth due to its large number of output channels. Some ducts are lined with distinct striped basal epithelium and microvilli. This describes which of the following excretory ducts? A. *Striated ducts

83 B. Excretory ducts C. Intralobular ducts D. Intercalated ducts E. Interlobular

Many small excretory ducts of the salivary gland open onto the surface of the tongue, a topographic location which is not typical for the glands. Specify the location of the salivary gland of the tongue? A. *Muscularis mucosae B. Mucous membrane C. Submucosa D. Epithelium of the tongue superior surface E. Epithelium of the tongue inferior surface

Due to medical reasons, one of the salivary glands was removed from a 47-year old patient, after which the amylase in saliva sharply decreased. Which gland was removed? A. *Parotid B. Buccal C. Submucosa D. Gingival E. Sublingual

In the parotid salivary glands rapid sclerosis of its parenchyma and decreased production of its biologically active hormonal substances began as a result of a viral process. Because of this the regeneration of oral mucosa is damaged. What is missing in the saliva? A. *Growth factors of epithelium B. Interstitial cell-stimulation factors C. Colony-stimulating factors(CSF) D. Lysozyme E. Parotin

Epithelium of the interlobular excretory ducts is observed as a result of chronic inflammation of the parotid gland. Which epithelium is damaged? A. *Slightly stratified epithelium B. Stratified columnar C. Simple squamous D. Stratified ciliated E.

The parotid glands of the secretory department are damaged due to acute inflammation. As a result of this, which cells will suffer? A. *Serous and myoepithelial cells B. Serous and mucosus C. Myoepithelial D. Serous and mucosus E. Mucosus and myoepithelial cells

In a specimen of a removed salivary gland we find a small stone in a duct, the wall of which is lined with stratified squamous epithelium. In which duct of the gland is this duct located? A. *Excretory ducts B. Interlobular C. Striated D. Intercalated E. Intercellular

84 To determine the localization of pathological process in the palatine, the dentist uses all of the following terms to describe areas of the hard palate except: A. *Alveolar(acinar) zone B. Glandular C. Adipose D. Marginal zone E. Palantine raphe

Stones are formed in the submandibular gland of a patient which has a saliva disease. What is the structure of this gland? A. *Compound branched-tubuloacinar B. Compound nonbranched tubulo-acinar C. Simple branched tubulo-acinar D. Compound branched acinar E. Simple nonbranched tubular

Underdeveloped Dzhianutsi's cresents are found in a histological specimen of the submandibular salivary gland. Which type of secretion prevails in this gland? A. *Mucous B. Serous C. Endocrine D. Mixed E. Eccrine sweat

A patient has reduced secretion of saliva enzyme component. The acini of which cells of the salivary gland were damaged? A. *Serous cells B. Mucous cells C. Endocrine cells D. Myoepithelial cells E. Epithelial cells of intercalated ducts

The secetory ducts were damaged as a result of acute inflammation of the salivary glands. Which cells are affected by this? A. *Serous, mucous and myoepithelial cells B. Mucous and myoepithelial cells C. Serous cells D. Myoepithelial cells E. Serous and myoepithelial cells

Liver.

On histological preparation parenchyma of organ it is submitted by lobes which have the 6-angular prisms shape and consist from trabeculas between sinusoidal capillaries which radiality converge to the central vein. What anatomic organ has the given morphological structure? A. Тhymus B. Pancreas C. Liver * D. Spleen E. Lymph node

There is large number of carbohydrates in the human diet. What structures are detected while in the cytoplasm of hepatocytes? A Glycogen granules*

85 B Droplets of fat. C One big fat drop. D Increase the number of free ribosomes. E The inclusion of lipofuscin.

A viral infection has damaged cells that form walls of bile capillaries. This stimulated conditions for inflow of bile into the blood of sinusoidal capillaries. What cells are damaged? A Hepatocytes * B Kupffer's cells C Ito cells D Pit-cells E Endotheliocytes

During proliferation of connective tissue in parenchyma of the liver (fibrosis) due to chronic diseases there is disruption of blood circulation in classic liver lobule. What is the direction of blood flow in these structures? A * From the periphery to the center B From the center to the periphery C Around lobule D From the top to the bottom E From the bottom to the top

Examination of 28-year-old patient with hepatocerebral degeneration revealed an impairment of ceruloplasmin synthesis. This defect is associated with the following organelles: A. *Granular endoplasmic reticulum B. Smooth endoplasmic reticulum C. Mitochondria D. Lysosomes E. Golgi complex

An electron micrograph of internally lobed liver sinusoid represented cells, which can be seen in the cytoplasm granules with a seal like fruit pit. We know that this is a natural killer cell. Which cell is represented. A * Pit-cell B Hepatocytes C Endothelial cells of sinusoid D Kupfer cells E Fat-storing cell

In the patient from therapeutic department with severe liver pathology revealed violations of coagulation. What is the function of the liver can be affected in this case? A * Protein Synthesis B Detoxication C Endocrine D Protective E Bile formation

An examination of the patient revealed abnormalities of the liver. Which embryonic germ layers have been damaged? A * Endoderm middle sections of primary colon B Endoderm posterior wall C Foregut endoderm D Mesonephritic duct E Hindgut endoderm

86

As a result of stab wounds in the liver was damaged the hepatic artery, but the liver lobule blood continued to flow. Which vessel provided blood flow in lobules? A * Interlobular vein B Interlobular artery C Portal vein D Sublobular vein E Hepatic vein

In histological slide of parenchyma of the organ slices is presented, the shape is hexagonal prisms and consist of anastomosing plates between which are sinusoidal capillaries that converge radially to the central vein. What is the anatomical organ has given morphological structure? A * Liver B Pancreas C Thymus D Spleen E Lymph node

During carbohydrate over-feeding of the animals in the cells of the liver we can find histologically a large amount of glycogen granules. What group of cell structures includes glycogen? A * Trophic inclusion B Secretory inclusion C Excretory inclusion D Pigment inclusion E Organelles with special functions

As a result of hepatotrophic poisons in hepatocytes was destroyed granular EPR. Synthesis of which components will be change in the epithelium of the liver? A * Albumin and fibrinogen B Phospholipids C Glycogen D Cholesterol E Vitamins

Damage of junctions between liver hepatocytes which causes pathological processes as a result of it bile gets into bloodstream, causes jaundice. Disorder of what type of intercellular junction may explain this appearance? A. *Tight (zonular occludentes) B. Simple contact C. Zonular adherentes D. Desmosomes E. Gap junctions

A large number of glycogen is detected in the cytoplasm of a hepatocyte in a specimen. What process in the body causes this appearance? A. *Increased blood sugar B. Reduced blood sugar C. Increased absorption of lipids in the intestine D. Reduced absorption of lipids in the intestine E. Increased absorption of proteins in the intestine

A patient with signs of jaundice is admitted into the surgery department. Bile from the bile capillary does not enter the bloodstream in normal circumstances. What ultrastructural feature of hepatocyte structure contributes to this?

87 A. *Presence of zonula occludentes between hepatocytes B. Their polygonal shape C. Presence of hepatocyte biliary surface D. The absence of bile capillary wall E. Presence of microvilli on the surface of the capillary

A polygonal cell with a bright big nucleus and big nucleolus is found in microscopic research of a liver lobe. It has many well developed organelles and inclusions in it's cytoplasm. What are these cells? A. *Hepatocytes B. Endothelial cells C. Stellate macrophage D. Pit-cell E. Ito-cell

In a specimen of parenchymal organ we can make out unclearly distinct hexagonal shaped segments, in the centre of which lies a vein and in interlobular tissue are the triads(artery, vein, and excretory ducts). What organ is this? A. *Liver B. Pancreas C. Thymus D. Spleen E. Thyroid gland

The walls of bile capillaries are damaged as a result of a viral infection. This created conditions for flow of bile into the blood of the sinusoidal capillaries. What cells were damaged? A. *Hepatocytes B. Fat-storing cells C. Ito-cells D. Pit-cells E. Endothelial cells of sinusoid

Pancreas.

The patient since 14 years old has diabetes. What endocrine cells of pancreatic islands don’t function? A. D - cells B. A - cells C. D1-cells D. B - cells * E. РР - cells

In patients after acute pancreatitis is determined massive damage of acinar cells. By which cells will go their recovery? A Intercalated duct cells* B Cells of islets of Langerhans C Cells of interlobular duct D Cells of gland’s stroma E Endothelium of blood vessels

Endocrinologist diagnosed in patient disorders of the endocrine function of the pancreas, resulting descending of the hormone glucagon in the blood. The function of this gland cells broken in this case? A A-cells of the islets of Langerhans* B B-cells in the islets of Langerhans C D-cells of islets of Langerhans D D1-cells of islets of Langerhans

88 E PP-cells of islets Lanhenhansa

A ptient has disturbed digestion of proteins, fats and carbohydrates. It is most licely to be caused by reduced secretion of the following digestive juice: A. 8Pancreatic B. Saliva C. Gastric D. Bile E. Intestinal

A 50 years old patient complains with increased appetite, thirst, decreased body weight, weakness and fatigue. During laboratory examination revealed increase the amount of sugar in the blood. Which cells dysfunctions are associated with the development of this disease? A * B cells B A -cell C Thyrocytes D Pankreatocytes E Lipotropocytes

In histological slide were studied exocrine portion of the pancreas. In the cells of the exocrine parenchyma contained secretory granules with enzymes. How do they come (these enzymes) to the digestive tract? A * Through duct system B Through the bloodstream C Fall into the lymph D Aхonic transport E Dendritic transport

During the study of pancreatic cells plasmolemma after exposure to drugs offenses were discovered in the structure of the glycocalyx. What are the chemical components of the glycocalyx is composed of cells of the pancreas? A * Oligosaccharides B Proteins C Lipids D Mineral salts E Water

In the histological slide of the pancreas we can find a group of cells. Some of them are centrally located and has basophilic secretory granules. Their secret regulates carbohydrate metabolism. What kind of cells they are? A * B cells B PP cells C A cells D Adipocytes E D cells

A high concentration of insulin is revealed in the blood of a 48-year old woman. The activity of which pancreatic cells causes this? A. *B-cells B. A-cells C. D D. D1 E. PP

89 In a preparation of a gland we distinguish acini, which consist of 8-12 secretory cells with a cranial shape, has a light nucleus and 1-2 nucleoli.The basal part of the cell is stained basophysically and the apical part contains oxyphyllic granules, myoepitheliocytes are absent. What is this gland? A. *Pancreas B. Parotid glands C. Submandibular salivary gland D. Breast E. Oil bag

Crypts of the small intestine are composed of all the following except: A. *Kupfer's cells B. Goblet cells C. Panet's cells D. Intestinal epithelial cells E. Columnar epithelial cells

A 55-year old patient visits an endcrinologist about the violation of endocrine pancreatic function, manifested by a decrease of glucagon hormone in the blood. The function of which cells is damaged? A. *A-cells B. B-cells C. D-cells D. D1-cells E. PP

Respiratory system.

At the patient with a dry pleuritis noise of friction of pleura is listened. At what defeat of type of epithelia this symptom is marked? A. Compound B. Simple squamous* C. Transitional D. Cuboidal E. Cilindrical

In a histologic preparation the organ which wall consists of mucosa, submucosa, fibro-cartilage and аdventitial shalls is submitted. Epithelia - pseudostratified, lamina muscularis is absent, in submucous – mixed glands, hyaline cartilage forms the open-ended rings. What organ has this morphological attributes? A. Trachea* B. Terminal bronchiole C. Segmental bronchi D. Alveola E. Larynx

In an electronic microphoto the cells of alveoles are founded. They are the part of aero-haematic barrier. What are these cells? A. Secretory alveolocytes B. Respiratory alveolocytes * C. Alveolar macrophages D. Endocrinocytes E. Microphages

It is known, that the important component of aero-haematic barrier is surfactant which warns closed alveoli during breathe in. By what cells of alveoli are synthesized phospholipids which constructed these

90 membranes? A. Alveolocytes ІІ type * B. Respiratory cells C. Goblet cells D. Alveolar macrophages E. Endotheliocytes of capillaries

In patient with laryngeal diphtheria doctor founded membranous coats on mucosa, that strong connected with epithelia. To what types of epithelia, marked below, the membranous coats fixed strong? A. Compound keratinazed B. Compound non-keratinazed * C. Pseudostatified D. Simple sqamous E. Simple cilindrical

In histologic preparation of the organ which wall has mucous, submucous, fibrous-cartilage and adventitial shall is submitted. Epithelia - pseudostratified. In submucous – end-pieces of the mixed glands. Hyaline cartilage forms large plates. What organ has the given morphological attributes? A. Large bronchi * B. Trachea C. Small bronchi D. Larynx E. Middle bronchi

In an electronic microphoto of biopsy structures into which structure enters surfactant, alveolocytes I type, basal lamina and fenestrated epithelia of capillaries are submitted. What histo-haematic barrier in human body content from these structures? A. Haemato-encefalic B. Haemato-thymic C. Aero-haematic * D. Uro-haematic E. Haemato-testicularis

The patient has acted arrived in branch with attack of asthma which is caused by spasm of smooth muscles of respiratory ways. Name departments of respiratory ways to which basically it is connected this attack. A. Bronchi of small caliber* B. Bronchi of middle caliber C. Bronchi of large caliber D. Terminal bronchioles E. Respiratory departments

Lung of premature infant is presented on electronic photomicrography of biopsy material. Collapse of the alveolar wall caused by the deficiency of surfactant was revealed. Disfunction of what cells of the alveolar wall caused it? A Alveocytes type II * B Alveocytes type I C Alveolar macrophages D Secretory cells E Fibroblasts

A patient was admitted to the hospital with an asphyxia attack provoked by a spasm of smooth muscles of the respiratory tracts. This attack was mainly caused by alterations in the following parts of the airways:

91 A Small bronchi * B Median bronchi C Large bronchi D Terminal bronchioles E Respiratory part

At electronic microphotography of pulmonary alveole's wall a big cell presents. Its cytoplasm has a lot of mitochondria, developed Golgi apparatus, osmiophil lamellated corpuscles. What is the main function of this cell? A It produces surfactant * B It is a component of blood-air barrier C It warms the air D It purifies the air E It absorbs microorganisms

A pathological process in bronchi caused desquamation of epithelium. What cells will regenerate bronchial epithelium? A Basal * B Intercalary C Ciliate D Endocrinal E Goblet

Alveolar space of the acinus was invaded by some bacteria which interacted with the surfactant.This led to the activation of the cells that are localized in the alveolar walls and on the surface. What cells are these? A Alveolar macrophages * B Alveolocytes type I C Endothelial cells D Clara cells E Alveolocytes type II

A histological specimen represents an organ whose wall consists of the mucosa, submucosa, fibrocartilage and adventitious cartilage. The organ is lined by pseudostratified ciliary epithelium, the muscular layuer of mucosa is absebt, the submucosa contains seromucous glands. Hyaline cartilage C- rings are present. What organ has described morphological characteristics? A. *Trachea B. Bronchiole C. Secondary bronchus D. Terminal bronchiole E. Larynx

During dissection (autopsy) of the 65 years old man, who suffered from lung disease pathological process was mainly localized in the bronchi, where during the histological examination were clearly visible glands, cartilage islands and pseudostratified epithelium. Which bronchi were changes? A * Secondary bronchi B Primary bronchi C Tertiary bronchi D Bronchioles E Terminal bronchioles

In electronic microphotograph were detected cells of the alveoli, which are part of the blood-air-barrier (aero-hematic barrier). What are these cells? A * Respiratory epithelial cells of the alveoli

92 B secretory epithelial cells of the alveoli C Alveolar macrophages D Clara cells E Ciliated epithelial

In 66 years old man was diagnosed a malignant epithelial tumor, originating from the medium-sized (secondary) bronchi. What epithelium is the source of this tumor? A *Pseudostratifed B Stratified squamous nonkeratinized C Stratified squamous keratinized D Simple columnar E Transitional

In the preterm baby were found sticking walls of the alveoli due to lack of surfactant. Please indicate dysfunction of which cells of the wall of alveoli causes this condition. A * Alveolocytes type II B Fibroblasts C Alveolocytes type I D Alveolar macrophages E Secretory Clara cells

In the alveoli of the lungs are present special cells through which gas exchange, they are part of the aero- hematic barrier. What are these cells? A * Alveolocytes I type B Clara cells C Alveolar macrophages D Alveolocytes II type E Epitheliocytes with microvilli

As a result of trauma 30 years old man’s nose damaged mucous membrane that covers the upper part of the nasal cavity. What will be the result, what it led to? A * Imperception odorous substances B Violation of humidification C Violation of secretory activity of goblet cells D Violation of warm air E Violation of warming and humidification

The patient has an inflammation process in the trachea, which captures the epithelium and lamina propria of the mucosa. What epithelium has undergone a change? A * Single layer ciliated B Stratified squamous nonkeratinized C Stratified cuboidal D Transitional E Simple cuboidal

In the biopsy of the mucosa of patients with bronchial asthma revealed a significant number of cells with multiple metachromatic granules. Name this cell? A * Tissue basophils B Macrophages C plasma cells D reticulocytes E fibroblasts

Premature infants have syndrom of respiratory failure. Failure of what aerohematic barriere component

93 underlies this pathology? A *Surfactant B Capillary endothelium C Basal membrane of endothelium D Basal membrane of alveolocytes E Alveolocytes

A patient with an acute rhinitis has hyperemia and excessive mucus formation in nasal cavity. What epithelial cells of mucous membrane have the intensified activity? A *Goblet cells B Ciliated cells C Microvillous cells D Basal cells E Endocrine cells

A patient was admitted to the hospital with an asphyxia attack provoked by a spasm of smooth muscles of the respiratory tracts. This attack was mainly caused by alterations in the following parts of the airways: A *Small bronchi B Median bronchi C Large bronchi D Terminal bronchioles E Respiratory part

Study of a tubular organ revealed that its median membrane consists of solid hyaline rings. What epithelium lines mucous membrane of this organ? A *Multinuclear prismatic ciliated B Monostratal prismatic glandular C Monostratal prismatic with a border D Multistratal squamous nonkeratinizing E Monostratal cubical

In the chemical industry workers after inhalation of corrosive vapors death occurred in ciliated epithelial cells of the bronchi. How the cells of the epithelium regeneration take place? A * Basal cell B Goblet cells C Endocrine cells D Ciliated cells E cilia-free cells (Clara cells)

In the airway epithelium are present dome-shaped cilia-free cells, which are placed on the surface of microvilli. There cytoplasm contains glycogen granules, lateral and apical Golgi complexes, elongated mitochondria, and a few secretory granules. What is the name of these cells? A * Clara cells B Goblet cells C Endocrine cells D Cells without border E Cambial cells ologiIn histological slide of the trachea are muco-ciliary comprising low cells have oval or triangular shape. Their tops they reach epithelial apical surface, in some cells occurs mitosis. What function do these cells? A * Is a source of regeneration. B Part of the muco-ciliary complex.

94 C Secretes mucus. D Secretes surfactant. E Producing bically active substances.

After a prolonged inflammation of the mucous membrane of the nasal cavity of a patient observed changes in the epithelium. What epithelium has undergone a change? A * Simple pseudostratified B Simple squamous C Stratified squamous D Stratified cuboidal E Stratified columnar

The mucosa is determined in histological preparation in the wall of the trachea. What epithelium is its composition? A *Ciliated pseudostratified columnar B Stratified squamous nonkeratinized C Stratified squamous keratinized D Simple columnar with brushed border E Glands

During an attack of asthma patient feels lack of air associated with spasm in one of the departments of airways. What is referred to? A *Small bronchoiles B Middle bronchoiles C Primary bronchi D Тrachea E Тerminal bronchioles

Flat cell with peripheral zone of cytoplasm appears in electron microphotograph of alveoli wall. What is the function of this cell? A *Is a component of air-blood barrier B Produce surfactant C Warms air D Filteration of air E Absorbs microorganisms

In the course of intubation tracheal wall was damaged. What layers will be damaged? A *Mucosa, submucosa, cartilagenous, adventitia B Mucosa, submucosa, muscularis, adventitia C Mucosa, cartilagenous, muscularis, adventitia D Mucosa, cartlagenous adventitia E Mucosa, submucosa, cartilaginous, serosa

The middle layer of tubular organ is formed by hyaline cartilage which forms incomplete rings during slide investigation. What organ is this? A *Тrachea B Primary bronchi C Secondary bronchi D Small bronchoiles E Terminal bronchoiles

It was determined that middle layer of a tubular organ consists of solid hyaline rings during investigation. What epithelium lines mucosa of this organ? A *Ciliated Pseudostratified columnar

95 B Simple columnar C Simple columnar with brushed border D Stratified squamous nonkeratinized E Simple cuboidal

The aero-hematic barrier contains a surfactant, which plays a protective role and reduces surface tension. Determine the cells which forms the surfactant? A. *Type II alveolar cells B. Clara cells C. Brush cells D. Alveolar macrophages E. Type I alveolar cells

Premature infants often have pneumonia because alveolocytes type II (secretory) do not perform their function. What function should be performed by secretary alveolocytes? A. *Synthesis of surfactant membranes B. Phagocytosis C. The synthesis of enzymes which destroy surfactant D. Synthesis of hormones E. Respiratory

Chronic rhinitis is accompanied by metaplasia of the nasal cavity mucosal epithelium. Which of the epithelium types mentioned below is nasal cavity mucosa lined? A *Ciliated Pseudostratified columnar B Simple cuboidal C Stratified squamous keratinized D Stratified squamous nonkeratinized E Simple squamous

Urinary system. Kidneys. Urinary Tract.

The patient with disease of kidneys which is accompanied by an ischemia of parenchyma has the high level of arterial pressure. What conducting leading factor is the reason of increase the AP at this patient? A. High level of angiotensin ІІ* B. High level of vasopressine C. High level of renine D. Increase of sympathetic nervous system’s activity E. Hyperkateholemia

At the patient after a heavy trauma the shock has developed and attributes of acute renal insufficiency (ARI) have appeared. What is conducting leading mechanism of development ARI in this case? A. Stasis of urine B. Decreasing of blood pressure* C. Increasing of blood pressure D. Renal injury E. Nephroptosis

Symptoms of anemia are found out in the patient with chronic glomerulonephritis. What has caused occurrence of these symptoms? A. Decreasing of erithropoietines’ synthesis* B. Haematuria C. Increasing of normal erythrocytes’ destruction D. Haemolysis of erithricytes E. Decreasing of synthesis of hemoglobin

96

At the patient of 18 years at laboratory analyses glucose presence in urine is revealed at normal its concentration in blood plasma. The most authentic reason of it is infringement: A. Glomerular filtration B. Tubular secretion C. Glucocorticoids’ secretion D. Secretion of insulin E. Tubular reabsorbtion*

At electronic microscopy in renal cortex structures that lining by cylindrical epithelia with brush border and plicaes of basal plasmalemma are defined determined. Between plicaes there are many mitochondrias. What department of nephrons possesses the described structures? A. Henle’s loop B. Proximal convoluted tubule* C. Renal corpuscle D. Collecting tubule E. Capsula of nephron

At the girl of 5 years old, who was treated for cystitis, increase temperature up to 390 С is marked. On the basis of clinical and laboratory analyses the acute inflammation of renal pelvises is diagnosed (pyelonephritis). What structural features have most likely caused propensity of child to pyelonephritis? A. Renal infarct B. Undevelopment почечных лоханок C. Short ureters* D. Renal ectopia E. Small diameter of ureters

Developmental anomalies of urinary system meet, on the statistical data, in 10-14 % of newborn children. From which sources, marked below, kedney developmented? A. Splanchnotome B. Mezenchyma C. Nephrogonotome* D. Segmental legs E. Allantois

During clinical inspection in the 35-years woman with disease of kidneys in urine blood cells are found out, that is authentically connected to infringement of renal filter. This filter will consist of what structures? A. Endotheliocytes, podocytes B. Three-layered basal lamina C. Endotheliocytes, basal lamina D. Podocytes, basal lamina E. Endotheliocytes, basal lamina, podocytes *

In experiment increase of blood pressure is received from an animal by narrowing of renal arteries. What function of kidneys’ cells causes this effect? A. Podocytes B. Juxtaglomerular cells * C. Endotheliocytes D. Interstitial cells E. Cells of macula densa

At the laboratory analysis of urine of the patient it is revealed, that it has subacidic reaction. What cells of kidneys provide this reaction of urine?

97 A. Cells of collecting tubules * B. Juxtaglomerular cells C. Juxtavascular cells D. Cells of macula densa E. Interstitial cells

On preparation it is well visible capillary rete, located between two arterioles (rete mirabile). In what body it is possible to find out this picture? A. In kidney * B. In liver C. In suprarenal gland D. In spleen E. In lungs

The patient of 46 years has addressed to the surgeon with complaints to allocation from umbilicus a transparent liquid with unpleasant smell. At laboratory research of this liquid it is determined, that it is urine, the diagnosis is established. Surgical treatment is recommended. What structure it is necessary to lead carry out to the given patient? A. Urahus* B. Ureter C. Uretry D. Umbilical vein E. Umbilical cord

The low specific gravity of the secondary urine (1002) was found out in the sick person. What is the most distant part of nephron where concentration of secondary urine takes place? A In the collecting tubules * B In the nephron’s glomerulus C In proximal tubule of nephron D In ascending part of loop of Henle E In distal tubule of nephron

A histological specimen of a kidney shows a part of the distal tubule going between the afferent and efferent arterioles. The cells building the tubule wall have dense nuclei; basal membrane is absent. Such structural formation is called: A Macula densa * B Juxtaglomerular cells C Mesangial cells D Juxtavascular cells E -

A histological specimen of kidney shows a structure consisting of a glomerulus of fenestrated capillaries and a bilayer epithelial capsule. Specify this structure: A Renal corpuscle * B Proximal tubule C Distal tubule D Henle's loop E Receiving tube

In patients with chronic pyelonephritis there is no acidification of urine, so it is no bactericidal. Which structures of the kidney injury occurred? A * Dark cells of the collecting tubules B In the glomeruli C In podocytes capsules

98 D In the proximal convoluted tubules E In the distal tubules

Malformations of the urinary system occur, according to statistics, 10-14% of infants. What are the sources of the development of the urinary system? A * Nonsegmental caudal mesoderm B Dorsal mesoderm C Visceral leaf of ventral mesoderm D Parietal leaf of ventral mesoderm E Germ mesenchyme

The patient was suffered from edema (swelling). A large amount of protein present in urine. About dysfunction of which part of nephron does it mean? A * Renal corpuscle B Proximal convoluted tubule C Distal convoluted tubule D Downturn of the loop of Henle E Ascending part of the loop of Henle

Microscopic study of kidney biopsies found in its cortex tubules about 60 microns in diameter; their wall is formed by a high cuboidal epithelium with abundant long microvilli. What structure is it? A * Proximal tubules B Distal tubules C Capsule kidney cells D Team rolls E Loop of Henle

In histological slide of renal cortex can be seen renal corpuscles and tubules of nephrons. It is known that in the tubules of the nephron reabsorption occurs. Which tissue of the nephron involved in this process ? A * Epithelial B Connective proper C Reticular D Mucous E Cartilaginous

An important part of the renal filtration barrier is a three-layer basement membrane, which has a special network structure of the middle layer. Where is this basement membrane? A * In the renal corpuscles B In capillaries of the perytubular capillary network C In the proximal tubules D In the small tubules E In the distal tubules live

Electron-microscope investigation of cortical substance of a kidney reveals some structures lined with prismatic epithelium that normally has brush border and deep plicae of plasmolemma in its basal part. There is a big number of mitochondrions between these plicae. These structures belong to the following part of a nephron: A *Proximal tubule B Distal convoluted tubule C Henle's loop D Renal corpuscle E Distal straight tubule

99 In electron microscopy revealed the kidney tubules, which are lined by cuboidal epithelium. In the epithelium distinguish light and dark cells. Light cells had organelles. These cells provide the reabsorption of water from primary urine in the blood. Dark cells in structure and function resemble parietal cells of the stomach. What tubules represented in microscopic slide? A *Collecting ducts B Proximal convoluted tubules C Distal tubules D Ascending part of the loop of Henle E Descending part of the loop of Henle

During the clinical examination in 35 -year-old women with kidney disease identified in urine blood cells, fibrinogen, which is likely due to the renal filter. Which structures are composed this filter? A * Endothelium of the capillaries of the glomerulus, three-layer basement membrane, podocytes B Three basement membrane C Endothelium of the capillaries, the basement membrane D Podocytes, the basement membrane E Endothelium, podocytes

In a histological slide of the kidney there is a defined structure consisting of glomerular capillaries and epithelial bilayer capsule. Identify the structure? A *Renal corpuscle B Proximal tubule C Distal tubule D Loop of Henle E Collecting tubules

A laboratory analysis of urine of a patient hospitalized with glomerulonephritis revealed altered erythrocytes. What part of the nephron was the most damaged? A *Bowman's capuscle B Proximal convoluted tubule C Distal convoluted tubule D Loop of Henle E Collecting tubules

A patient ill with chronic glomerulonephritis has a disturbed incretory function of kidneys. It will result in the deficit of the following blood corpuscles: A *Erythrocytes B Leukocytes C Thrombocytes D Leukocytes and thrombocytes E Erythrocytes and leukocytes

A microphotography represents a fragment of cortical substance of a kidney. This fragment contains thick spot cells and juxtaglomerular cells with big secretory granules. What kidney structure is represented? A *Juxtaglomerular apparatus B Renal corpuscle C Filtering barrier D Prostaglandin apparatus E Choroid glomus

Urine was taken from 17 year old young man from a bladder using the catheter. Which epithelial cells of the urinary bladder can be detected by microscopy? A * Transitional

100 B Stratified nonkeratinized C Simple columnar D Simple cuboidal E Stratified keratinized

A histological specimen of a kidney shows a part of the distal tubule going between the afferent and efferent arteriole. The cells building the tubule wall have dense nuclei; basal membrane is absent. Such structural formation is called: A *Macula densa B Juxtaglomerular cells C Mesangial cells D Juxtavascular cells E -

A female patient with pyelonephritis was admitted to the urological department. Examination revealed an associated infection accompanied by pyelovenous reflux. This complication was induced by affection of the following structure: A *Fornical renal apparatus B Excretory renal tracts C Straight tubules D Renal tubules E Renal corpuscle

In course of an experiment the blood pressure of an animal had a stable rise by means of renal artery constriction. Hyperfunctioning of what renal cells cause this effect? A *Juxtaglomerular cells B Podocytes C Endotheliocytes D Interstitial cells E Thick spot cells

Electron micrograph of a kidney fragment presents an afferent arteriole with big cells under endothelium. These cells contain secretory granules. Name this type of cells: A *Juxtaglomerular B Mesangial C Smooth muscular D Juxtavascular E Interstitial

Hypertension is observed in a 28 year old man after kidney illness. This may be due to? A *Disorder of excretion of urine B Disorder of excretion of aldosterone C Disorder of excretion of throxine D Disorder of excretion of testesterone E Disorder of excretion of somatotropin

The mucous membrane of the tubular organ is lined with transitional epithelium which forms longitudinal folds. The muscle sheath consists of two layers in the upper half and three at the bottom. What organ is this? A *Urethra B Urinary bladder C Esophagus D Rectum E Uterine tubes

101

While observing a sick patient with a tumor in his bladder, the doctor performed a cytoscopy. On examination of the mucosa, he noticed an area without folds, which part of the bladder is this phenomenon characterized? A *Fundus B Body C Neck D Apex E Tunica mucosa

A sick 35 year old patient went to the clinic with complaints of heart burn while urinating and unpleasant developments in the perineum. After examination, he was diagnosed with inflammation of the bulbo-urethral gland and as a result, a decreased secretion of these glands and irritation of the urethral mucous membrane. In which part of the urethra do these excretory ducts open? A *Cavernosus B Membranosus C Prostatic portion D Spongiosum E

Male reproductive system. Testis.

In preparation of organ with many tubules which wall is formed by own environment which will consist from basal, myoid and fibrous layers is submitted. On basal lamina supporting cells and spermatogenic epithelia placed. What organ is submitted in a preparation? A. Epididimis B. Testis* C. Ductus deferens D. Ejaculatory duct E. Prostata

In an embryo on 2-3 week of development are found out gonocytes - predecessors of reproductive cells. In what material these cells are differentiated? A. In dermatomes B. In mesenchyma C. In germinal ectoderm D. In yolk sac * E. In germinal endoderm

At the man of 35 years after the received doze of an irradiation it is broken spermatogenesis. From what cells there is a restoration of spermatogenesis? A. Glandulocytes B. Spermatozoa C. Spermatogonia* D. Sustentocytes E. Spermatides

At the middle-aged man occurrence is observed secondary sexual attributes behind female type. What function of testis’ cells is reduced? A. Sustentocytes B. Glandulocytes* C. Spermatozoa D. Exocrinocytes E. Fibrocytes

102

In wall of yolk sac cells with the large nucleus, the raised increased maintenance contents of glycogen and high activity of alkaline phosphotaze are found out. Subsequently these cells with blood on vessels migrate to mesonephros. In what cells of man's reproductive system they are differentiated? A. Leidig’s cells B. Supporting cells C. Spermatogenic cells * D. Fibroblasts E. Blood cells

Activity of man's reproductive system is adjusted by nervous and endocrine systems? What from the listed hormones adjust activity of man's reproductive system? A. Glycocorticoids B. Gonadotrophins C. Thyreotroph hormon D. Теstosterone* E. Epinephrine

At electronic microscopy coiled seminiferous tubules’ cells of pyramidal shape have been found out. In cytoplasm of these cells it is well advanced smooth endoplasmic reticulum, Golgy complex and various inclusions. What cells are there? A. Spermatogonii B. Leidig’s cells C. Sustentocytes * D. Fibroblasts E. Gonocytes

Attributes of cryptorchism are found out in the boy – undescendence testis in scrotum. If not to perform operation, it can result in what consequences? A. Death of the supporting cells B. Destruction of coiled seminiferous tubules’ wall C. Cessation of spermatogenesis* D. Migration of testis E. Death of the glandulocytes

At research of prostatic liquid at the patient of 25 years the insufficient quantity amount of reproductive cells is revealed. From what cells usually provide sufficient quantity amount for fertilisation of spermatozoa? A. Spermatogonia * B. Sustentocytes C. Supporting cells D. Leidig’s cells E. Glandulocytes

During pubescence the cells of male sexual glands begin to produce male sex hormon testosterone that calls forth secondary sexual characters. What cells of male sexual glands produce this hormone? A Leidig cells * B Sustentocytes C Sertoli's cells D Supporting cells E Spermatozoa

One of the phases of spermatogenesis observed changes in the nucleus and cytoplasm of spermatids, leading to the formation of mature gametes. What phase of gametogenesis is it?

103 A * Formation. B Maturation. C Growth D Reproduction E Proliferation

In testes destructive changes observed in cells of seminiferous epithelium, which are already in the stage of formation - spermatids and spermatozoa. By which cells can restore spermatogenesis? A * Spermatogonia B Primary spermatocytes C Secondary spermatocytes D Supporting cells E Leydig cells

In histological slide of the testes in layers of connective tissue between the seminiferous tubules occur as vascular nests of pale acidophilic cells with well-developed smooth endoplasmic reticulum, Golgi complex and mitochondria. What kind of cells they are? A * Interstitial cells B Supporting cells C Myoid cells D Fibroblasts E Spermatogonia

During puberty, the cells of male gonads begin producing the male sex hormone testosterone, which causes the appearance of secondary sex characteristics. Which cells of male gonads produce this hormone? A * Leydig cells B Sustentocytes C Sertoli cells D Supporting cells E Spermatozoa

A microscopic examination reveals cells of seminiferous tubules undergoing meosis. Determine he stage of spermatogenesis these cells belong to? A *Maturation B Considerable growth C Little growth D Formation E Reproduction

During an experiment the Leydig cells of the testis were damaged. What changes can be observed in the blood which flows from the testis? A *The level of testesterone will decrease B The level of testesterone will increase C The level of inhibitin I will decrease D The level of inhibitin II will increase E the level of inhibitin III will decease

Some drugs have the ability to permeate through the blood testis barrier. Which of the following structures is included in the barrier? A *Blood capillaries endotheliocytes B Interstitial endocrynocytes C Spermatoblasts D Primary spermatocytes

104 E Secondary spermatocytes

Seminiferous tubules with seminiferous epithelium lining are observed in a histological preparation. Name the source of development of this epithelium? A *Genital cord mesoderm B Mesonephric duct epithelium C Neuroectoderm D Yolk sac ectoderm E Neural crests

Spermatogenesis consists of four phases What is a typical characteristic of the formation phase? A *It includes cytodifferentiation of spermatids B It is observed before puberty C It occurs in diploid cells D It is the result of spermatocyte production E It includes increased rate of germ cells

A 43 year old man experienced disturbance in his sperm due to his previous orchitis. Pathological changes occured in which part of the testis? A *Ductuli seminiferi contorti B Rete testis C Ductuli seminiferi recti D Ductuli efferentes testis E Ductus epididymis

A married couple complains about their inability to have children. An examination revealed that the man's spermatogenic epithelium of the testis was damaged which causing absence of sperm in seminal fluid and as a result infertility. Which part of the deferens duct was damaged? A *Seminiferous tubules B Straight seminiferous tubules C Rete testis D Ductuli efferentes

In a histological specimen of the cellular tissue in the testis that seperates the seminiferous tubules, we see round shaped cells with an oxyphylic cytoplasm. What are these cells? A *Leydig cells B Sertoli cells C Secondary spermatocytes D Primary spermatocytes E Spermatogonias

A mother of a three year old boy arrived in the hospital with the concerns of her sons enlarged scrotum. An investigation revealed the accumulation of fluid between the membranes of the testes causing hydrocele. This occurred to a connection between the tunica vaginalis of the testes and abdominal cavity. What tunica of the abdomen passes into the tunica vaginalis of the testes? A *Peritoneum B Lumbar fascia C Superficial fascia D Subcutaneous tissue E Skin

Male reproductive tract. Accessories glands. At the patient with a chronic prostatitis unfertilization is observed. What reason of it? A. Disturbance of spermatogenesis

105 B. Unspermatogenesis C. Disturbance of hormonal function D. Decreasing of mobility of spermatozoa* E. Disturbance of immunity system

Female reproductive system. Ovaries.

In the ovary specimen colored with hematoxylin-eosin, follicle is determined where cubic-shaped follicle epithelium cells are placed in 1-2 layers, and zona pellucida covering is seen around ovocyte. Name this follicle: A Primary* B Primordial C Secondary D Mature E Atretic

The body of unknown woman has been delivered to medicolegal examination. On section in ovary round formation in diameter about 5 sm that contains a pigment of yellow color is revealed. This formation will consist of what cells? A. Interstitial cells B. Myocytes C. Follikular cells D. Fibroblasts E. Luteocytes*

At preparation of ovary are determined structures which have the big cavity. Оvocyte in them it is surrounded with zona pellucida, zona radiata. Wall is formed by layer of follicular cells and teka. Specify what structure of ovary has given morphological attributes. A. Mature follicle * B. Premordial follicle C. Primary follicle D. Yellow body E. Atretic body

In blood of the woman have found out the increased quantity amount of oestrogens. What cells synthesize these hormones? A. Ovocytes B. Follicular cells of secondary follicles * C. Follicular cells of primary follicles D. Follicular cells of premordial follicles E. Follicular cells and ovocytes

In the woman 50 years old it is found out cyst of ovary. From what structure it has developed? A. From a follicle * B. Ovary cortex C. Atretic body D. White body E. Interstitial cells

On preparation of ovary many large blood vessels is revealed. What structure of ovary on preparation? A .Cortex B. Medulla * C. Thertery follicle D. Atretic follicle

106 E. Yellow body

At microscopic research of biopsy of endometrium women who suffers on unfertility, are found out changes in its structure, caused by action of progesterone. Where this hormone is produced? A. In hypothalamus B. In follicles C. In neurohypophisis D. In adinohypophysis E. In yellow body *

Normal human embryo implantation can occur only with the corresponding changes of endometrium. Which cells of endometrium will increase during this process? A * Decidual cells B Macrophages C Endothelial cells D Fibroblasts E Neurons

A female patient presents with the ovarian hyperemia, increased permeability of the blood-follicle barrier with the development of edema, infiltration of the follicle wall with segmental leukocytes. The follicle is large in volume. Its wall is sickened. What period of the sex cycle is the described situation typical for? A. *Preovulatory stage B. Ovulation C. Menstrual period D. Postmenstrual period E. Period of relative rest

In the histological slide of the ovary, stained with hematoxylin-eosin, were identified a follicle in which follicular epithelial cells located in 1-2 layers and have a cuboidal shape, around the oocyte membrane is visible bright red color. What is the name of this follicle? A * Primary B Primordial C Secondary D Mature (Graafian) E Atretic

A patient underwent Caesarean section. During the operation a long incision was made in the uterus wall and the fetus was extracted from uterus. Healing of the sutured myometrium will proceed in the following way: A *Formation of a fibrous cicatrix B Formation of smooth muscular tissue C Formation of cross-striated muscle fibers D Proliferation of myosatellitocytes E Hypertrophy of smooth myocytes

A 35 y.o. patient diagnosed with sterility came to gynaecological department for diagnostic biopsy of endometrium. Microscopic examination revealed that mucous membrane is edematous, uterine glands are convoluted and filled with thick secreta. Such changes in the endometrium are caused by excess of the following hormon: A *Progesterone B Estrogen C Testosterone D Somatotropin

107 E ACTH

In the cortex of the ovary histological examination determined mature tertiary follicle. In which period of oogenesis they were formed? A * Stage of great growth B Stage of small growth. C Maturation. D Reproduction. E Formation.

For early diagnosis of pregnancy in women using their urine. The presence of which hormones would likely indicate a pregnancy? A * Chorionic gonadotropin B Estriol. C Aldosterone. D Testosterone. E Progesterone.

As a result of inflammation in the fallopian tubes the moving of zygote was difficult. What type of epithelium was changed? A * Simple columnar ciliated B Stratified squamous nonkeratinized C Pseudostratified D Simple cuboidal E Stratified squamous keratinized

Microscopic examination of the ovary, which was removed during operation were detected primordial and primary follicles and corpus luteum in the stage of prosperity. In what kind of stage of ovarian- menstrual cycle are the female reproductive organs with these structural features of the ovary? A *Premenstrual B Menstrual C Preovulatory D Postmenstrual E Ovulatory

The 35 years old patient, diagnosed with infertility in the gynecological department, diagnostic endometrial biopsy was done. Microscopic examination revealed that the mucosa with edema, uterine glands are meandering, filled with thick secret. What hormone causes the following changes in the endometrium? A * Progesterone B Estrogen C Testosterone D Somatotropin E ACTH

Stop bleeding after childbirth is associated with the action of hormones on the structures of the uterus. Which component of the uterine wall takes part in this most of all? A * Middle layer of myometrium B Endometrium C The inner layer of the myometrium D The surface layer of the myometrium E Perimetrium

Histology of the endometrium is characterized by the following features: thickening, edema, presence of

108 tortuous glands with enlarged lumen that secrete large amounts of mucus, mitosis in cells not observed, in stroma are available decidual cells. What is the stage of the menstrual cycle corresponds to the described picture? A * Secretory (premenstrual or luteal). B Menstrual C Regenerative D Proliferative E Relative calm

Microscopic examination of the female internal reproductive organs, which were removed during operation was found an embryo constructed of two blastomeres. Name the place of its location in case of normal development. A *Fallopian tubes, close to the ampulla B Fallopian tubes, close to the uterus C The cavity of the uterus D Abdomen E Ovary

By producing a number of hormones placenta plays a part of temporary endocrine gland. What hormone may be detected in woman's blood on the 3rd or the 4th day after begin of implantation, that is used in medicine for early pregnancy detection? A *Chorionic gonadotropin B Somatostatin C Progesterone D Vasopressin E Oxytocin

At 40 years old woman weak labor associated with weak contractility of the myometrium. What do you need to inject for the help? A *Oxytocin B Hydrocortisone C Dexamethasone D Aldosterone E Prednisone

In ovarian-menstrual cycle changes occur endometrial glands. Which of the following types are appointed endometrium glands? A *Simple unbranched tubular B Simple branched tubular C Simple unbranched alveolar D Alveolar unbranched E Alveolar-tubular branching

In histological slide of the ovary presented rounded structure, glandular cells of which contain lipid droplets. Identify this structure. A * Yellow body B Primordial follicles C Primary follicle D Mature follicle E Atretic body

Stop bleeding after childbirth is due to the action of oxytocin on the uterus wall. Which layer of the uterus reacts to the action of this drug? A * Myometrium

109 B Endometrium C Perimetrium D Parametrium E Submucosa

In a histological slide of ovarian follicle, we determine a large cavity. A primary oocyte is located in the cumulus oophorus area. It is surrounded by a transparent zone and radiant crown. Name this follicle? A *Mature follicles B Secondary follicles C Primary follicles D Atretic follicles E Primordial follicles

The gynecologist noticed several miscarriages in a patients history. This is due to the failure of which hormone? A * Progesterone B Estrogen C Gonadotropin D Chorionic gonadotropin E Relaxin

After analyzing a woman's blood, it is observed that the amount of progesterone and estrogen are approaching low level. At what stage of the cycle was the blood taken? A *The menstrual phase B Postmenstrual phase (secretory) C Postmenstrual phase (proliferation) D Relative calm E Postmenstrual

During pregnancy the formation of milk process is inhibited, but the growth and differentiation of mammary gland alveoli increases. Which vitamin helps prepare for lactation? A *E B C C PP D B E D

After gynecological surgery a patient began to complain of vaginal discharge as well as pain and discomfort in the lower abdomen. An inflammation of the inner lining of the uterus was diagnosed after an examination. What layer was damaged? A *Endometrium B Myometrium C Perimetrium D Parametrium E Peritoneum

On the background of hormonal changes a patients process of blastocyst attachment to uterine lining was damaged. The lack of synthesis of which hormone caused these changes? A * Progesterone B Estradiol C Follicle stimulating hormone (FSH) D Testesterone E Somatotropin

110 The restoration of endometrium in the post-menstrual phase depends on the normal condition of the uterine glands. Which of the mention below types of epithelium are endometrial glands referred to? A *Simple unbranched tubular B Simple branched tubular C Simple unbranched alveolar D Compound unbranched alveolar E Compound branched tubulo-alveolar

Female reproductive system. Uterus. Mammary gland.

After gynecologic operation, the patient of 32 years old began to complain of pain in lower part of abdomen and profuse discharge from vagina. After inspection the diagnosis - an inflammation of an internal shall of uterus is established. What shall have suffered? А. Endometrium* В. Myometrium С. Perimetrium D. Parametrium Е. Peritoneum

At biopsy of mammary gland astra-shape cells located between basal lamina and lactocytes are found out. Named a source of development of these cells? A. Myotome B. Sklerotome C. Ectoderm * D. Dermatome E. Endoderm

In ovarian-menstrual cycle there are changes in endometrial glands. To what type these glands belong? A. Simple tubular * B. Simple tubulo-alveolar C. Simple alveolar D. Comound alveolar E. Compound tubular

Menstrual cycle.

In a preparation of ovary near to follicles of the different stadies of development appear atretic bodies and mature yellow body. What stage of menstrual cycle is answered with such condition? A. Premenstrual * B. Menstrual C. Postmenstrual D. Regenerative E. Growth of follicle

At the analysis of blood in not pregnant woman the age of 26 years finds out low concentration of oestrogens and high of progesterone. In what stage of ovarian-menstrual cycle the analysis of blood has been made? A. Premenstrual phase * B. Menstrual phase C. Postmenstrual phase D. Desquamation E. Proliferation

111